Thứ Ba, 28 tháng 4, 2015

Thời sự Y Khoa 04.2015

Thời sự y học số 370 – BS Nguyễn Văn Thịnh

1/ NỘI SOI (ENDOSCOPIE) : NHỮNG HỨA HẸN CỦA MỘT NGOẠI KHOA KHÔNG ĐỂ LẠI SẸO.
Những can thiệp với ống nội soi (endoscope) được đưa vào qua những lỗ tự nhiên đang phát triển mạnh nhưng còn phải vượt qua những khó khăn.
ENDOSCOPIE. ” Chọc thủng thành tiêu hóa khi can thiệp ngoại khoa vẫn còn được xem như là một biến chứng nghiêm trọng”, GS Marc Barthet, thầy thuốc ngoại khoa của Assistance publique des hôpitaux de Marseille-Aix Marseille Université, đã nhấn mạnh như vậy. ” Hôm nay, chúng tôi thực hiện động tác này một cách cố ý, trong mục đích cải thiện những kết quả của vài can thiệp ngoại khoa.”
Sau sự phát triển của chirurgie coelioscopique (ngoại khoa soi ổ bụng), cho phép mổ bên trong bụng bằng cách chỉ tạo những đường xẻ nhỏ, chirurgie endoscopique (ngoại khoa nội soi) qua những lỗ tự nhiên, được thực hiện không một đường xẻ nào trên da, cũng biểu hiện còn hơn một cuộc cách mạng đối với các bệnh nhân và những thầy thuốc. Kỹ thuật này mang lại hy vọng của một ngoại khoa không sẹo (chirurgie sans cicatrice), gây ít đau hậu phẫu hơn và cho phép phục hồi nhanh chóng. Tuy nhiên đó là một thay đổi vũ trụ thật sự mà những hứa hẹn khác thường không được che khuất những khó khăn. ” Cần phát triển những công cụ đặc thù cho phương pháp này “, GS Jean Escourroux, thầy thuốc chuyên khoa tiêu hóa thuộc bệnh viện đại học Toulouse đã nhấn mạnh như vậy. Sự nhiệt tình hết mực tiếp theo sự thành công của những can thiệp đầu tiên hôm nay đã nhường chỗ cho một sự phát triển có chừng mực, cho phép thăm dò từng bước tất cả những khả năng của lãnh vực mới này.
Hệ tiêu hóa, được mở ra bên ngoài ở hai đầu của nó, dĩ nhiên là lãnh vực áp dụng chủ yếu của phương pháp này qua những lỗ tự nhiên, mặc dầu ruột thừa đầu tiên được lấy ra bởi phương pháp này đi qua âm đạo của bệnh nhân. Thật vậy cửa vào này ít nhất có hai bất tiện, theo vài chuyên gia. Thật vậy qua âm đạo tạo nên một nguy cơ đối với những chức năng của cơ quan này, nhất là trên bình diện sinh dục, và phương pháp này loại trừ 50% population. Một con số không luôn luôn làm ngừng sự phát triển của vài điều trị, trong trường hợp này đối lại với những khả năng khác an toàn hơn cho các bệnh nhân nam…và nữ.
Dạ dày, thực quản và đại tràng vậy dường như là những ứng viên hứa hẹn nhất để can thiệp lên hệ tiêu hóa mà không cần chọc thủng da, nhờ những ống nội soi được đưa qua miệng hay hậu môn. Đó là những NOTES nổi tiếng, theo acronyme Anh (Natural Orifices Transendoscopic Surgery : ngoại khoa nội soi qua những lỗ tự nhiên ), rất được quảng bá bởi giới truyền thông nhưng còn phải vượt qua vài chướng ngại trước khi được phổ biến rộng rãi hơn.


Mối lo ngại trước tiên của các chuyên gia : nguy cơ nhiễm trùng, nguyên nhân của việc cấm vượt qua thành tiêu hóa. ” Nguy cơ đã tỏ ra không đáng kể đối với dạ dày, vốn chứa ít vi trùng, GS Barthet đã chỉ rõ như vậy. Một sự rửa bằng huyết thanh sinh lý cũng đủ tránh sự lây nhiễm. ” CO2, được sử dụng một cách hệ thống để tránh những embolie gazeuse, cũng dường như có một tác dụng kềm khuẩn. Ngoài ra, mặc dầu chưa có một công trình nghiên cứu nào xác lập mức nguy cơ đối với việc mở thành ruột, các chuyên gia báo cáo ít vấn đề liên quan đến nguy cơ nhiễm khuẩn. Những protocole kháng sinh liệu pháp khác nhau trước và sau can thiệp đang được đánh giá để còn làm giảm thêm nữa nguy cơ này.
Những chướng ngại khác thuộc về kỹ thuật và đòi hỏi sự tham gia của các nhà công nghiệp để tránh những công cụ mới. Thực hiện một đường khâu từ bên trong ống tiêu hóa, với phạm vi giới hạn dành cho ống nội soi, là một thách thức kỹ thuật. ” Những nhà công nghiệp, hy vọng một sự phát triển nhanh chóng của các kỹ thuật NOTES, tương tự với sự phát triển của nội soi bụng (coelioscopie), từ nay phần lớn đã từ bỏ “, một thầy thuốc ngoại khoa rất liên hệ trọng lãnh vực này đã kể lại như vậy.
Thật vậy, NOTES là một kỹ thuật hoàn toàn mới, trái với nội soi bụng (coelioscopie), vốn là một thích ứng của những kỹ thuật cổ điển. Tuy nhiên những phát triển đang được tiến hành, nhất là để biến đổi conception của những ống nội soi. Những ống này ngay khi chúng vượt qua con đường rất giới hạn của ống tiêu hóa phải được hướng một cách chính xác về cơ quan phải được điều trị. ” Từ nay chúng tôi có nhiều clip và stent để sữa chữa thành tiêu hóa, GS Barthet đã nhấn mạnh như vậy. Chúng tôi cũng sử dụng những prothèse để nối hai phần của ống tiêu hóa.”
(LE FIGARO 16/2/2015)
2/ NHỮNG KỸ THUẬT LAI GIỐNG.
” Khó khăn chính của các kỹ thuật NOTES là do sự kiện rằng ống nội soi chỉ có một axe de travail duy nhất, trong một hướng duy nhất “, GS Marc Barthet, thầy thuốc ngoại khoa ở Assistance publique des hôpitaux de Marseille-Aix Marseille Université đã nhắc lại như vậy. Vậy nhiều thầy thuốc ngoại khoa theo con được được mở ra bởi BS Tsin : một kỹ thuật lai giống (technique hybride) trong đó ống nội soi được hướng dẫn đi xuyên qua một incision coelioscopique. Những kỹ thuật này ngày nay chiếm phần lớn trong số 4.500 trường hợp NOTES được thống kê trên thế giới, mặc dầu những puriste cho rằng chúng không được mệnh danh như thế. Vào năm 2013, một sự thống kê những trường hợp châu Âu trong hai năm trước chỉ rõ rằng phần lớn các kỹ thuật được sử dụng là lai giống, những trường hợp cắt bỏ túi mật chiếm khoảng 8 can thiệp trên 10, tiếp theo là cắt bỏ ruột thừa.
Những kỹ thuật lai giống có thể là một giai đoạn trong bước đầu rèn luyện những kỹ thuật không gây sẹo. ” Từ những năm 1960, những ống nội soi cho phép thực hiện vài động tác điều trị, hôm nay bao gồm nhiều điều trị ngoại khoa “, GS Jean Escourroux, thầy thuốc tiêu hóa thuộc các bệnh viện đại học Toulouse đã nói rõ như vậy. Lấy đi vài polype, những thương tổn phẳng (lésions planes) trên niêm mạc, những sỏi mật, những u vùng của papille hay đặt những stent suốt đường tiêu hóa từ nay thuộc những cuộc mổ thường quy đối với các kíp lành nghề nhất.
Sự phát triển những công cụ mới, nhất là những clips de suture thích ứng với các ống nội soi, cũng đã cho phép các thầy thuốc chuyên khoa tiêu hóa sửa chữa những thương tổn của thành ruột và vượt qua giới hạn với ngoại khoa. Còn các nhà ngoại khoa, từng thường sử dụng những ống nội soi để được dẫn đường nhưng từ nay họ lợi dụng những chức năng điều trị của chúng nhưng trong một trường hoạt động một chiều, chứ không phải approche à angles multiples thông thường.
” Hai discipline hôm nay tìm ra một trường hoạt động chung và sự thiết đặt những đào tạo chung sẽ là cần thiết để phát triển những can thiệp nội soi qua những đường tự nhiên”, GS Marc Barthet đã nhấn mạnh như vậy.
(LE FIGARO 16/2/2015)
3/ NHỮNG KỸ THUẬT CÓ SỰ PHÁT TRIỂN CHẬM HƠN DỰ KIẾN
Cánh cửa đầu tiên hướng về ngoại khoa nội soi (chirurgie endoscopique) bằng những đường tự nhiên đã được mở ra vào năm 1998, trong hai hướng khác nhau. Vào năm đó, BS Daniel Tsin, một thầy thuốc sản phụ khoa Hoa Kỳ, khám phá một ruột thừa bị nhiễm trùng bằng cách thực hiện một soi bụng chẩn đoán phụ khoa (une coelioscopie de diagnostic gynécologique). Để tránh một lỗ mở qua da rộng hơn, ông quyết định lấy ruột thừa ra bằng một đường xẻ qua âm đạo của bệnh nhân. Sự thu hồi ruột thừa bị viêm thật mẫu mực và vị thầy thuốc mở rộng kỹ thuật lai giống (technique hybride) này vào việc lấy túi mật nhưng hai năm sau một ủy ban địa phương cấm ông tiếp tục những công trình của mình trong chiều hướng này. Phương pháp qua âm đạo, bằng nội soi đơn độc hay không, sẽ được tiếp tục bởi những người khác, chủ yếu đối với những can thiệp phụ khoa.
TRÁNH SỰ CUỒNG NHIỆT
Cũng năm đó 1998, BS Anthony Kalloo, thầy thuốc chuyên khoa tiêu hóa, trình bày quan niệm ngoại khoa nội soi qua những lỗ tự nhiên (chirurgie endoscopique par les orifices naturels) bằng cách vượt qua hàng rào tiêu hóa, về sau lấy tên bằng tiếng Anh là NOTES. Vào năm 2000 ông trình bày những kết quả đầu tiên trên heo và, vào năm 2003, trường hợp cắt bỏ ruột thừa đầu tiên trên người qua dạ dày được thực hiện ở Ấn Độ. Rất nhanh chóng, những uy ban được thành lập để chỉ đạo sự phát triển của các kỹ thuật NOTES nhằm tránh sự cuồng nhiệt ban đầu mà nội soi bụng (coelioscopie) đã gây nên và những thực hành xấu đã tiếp theo sau đó. Bắt đầu từ năm 2007, những trường hợp trên người tiếp nối nhau, ban đầu chủ yếu là những cắt bỏ ruột thừa và túi mật. ” Hôm nay, cuối cùng các kỹ thuật NOTES dường như không intéressant lắm đối với những chỉ định này, có thể được thực hiện với những kết quả rất tốt xuyên qua da và với những vết sẹo tối thiểu “, GS Marc Barthet thầy thuốc ngoại khoa của Assistance publique des hôpitaux de Marseille-Aix Marseille Université đã xác nhận như vậy “. Những chỉ định hứa hẹn nhất liên quan đến chính ống tiêu hóa, nhất là để lấy đi những khối u hay những phần của ống tiêu hóa sau đó phải sửa chữa.” Ông đặc biệt bảo về sự sử dụng phương pháp này đối với chirurgie bariatique, đặc biệt phức tạp và hôm nay còn gây nhiều biến chứng. Lãnh vực phát triển sau cùng của những phương pháp này nhằm sử dụng một ống nội soi để tạo một tunnel trong chính thành của ống tiêu hóa (môn vị, dạ dày, thực quản) để đi đến lớp cơ và lấy đi những u nằm sâu, khó có thể đạt đến. Khoảng 100 phẫu thuật thuộc loại này đã được thực hiện ở châu Âu.
Nếu sự quan tâm về phương pháp không gây seo đôi khi có thể tranh luận, ngoài khía cạnh thẩm mỹ, trong vài trường hợp, những ưu điểm được phát hiện ngay. ” Đứng trước những viêm tụy tạng cấp tính, khi phải đi đến và lấy đi khẩn cấp những mô hoại tử, những kỹ thuật NOTES đã cho phép giam một cách ngoạn mục ty lệ tự vọng có thể lên đến 50% với một phương pháp cổ điển hơn”, GS Jean Escouroux, thầy thuốc chuyên khoa tiêu hóa trong các bệnh viện đại học Toulouse đã nhấn mạnh như vậy.
(LE FIGARO 16/2/2015)
4/ TA CÓ THỂ GHÉP MỘT TRÁI TIM ĐÃ NGỪNG ĐẬP.
Các thầy thuốc ngoại khoa đã làm cho tim của một người hiến chết (donneur mort) sống lại rồi đã có thể ghép nó. Một kỳ công cho phép làm gia tăng số những mẫu ghép.
CARDIOLOGIE. Một cải thiện kỹ thuật được trắc nghiệm lần đầu tiên trên người ở Úc cách nay vài tháng, và mới đây ở Vương quốc Anh, hứa hẹn làm gia tăng rõ rệt số lượng tim có thể được ghép. Một enjeu có tầm cỡ vì lẽ nhiều bệnh nhân chết mỗi năm vì thiếu tim để ghép. Số những cơ quan có sẵn có thể nhảy vọt 25% nhờ kỹ thuật này, theo vài chuyên gia. Đó là một tiến bộ lớn của y học.
Trong một thời gian lâu, những cơ quan được ghép chỉ phát xuất từ những bệnh nhân bị chết não (mort encéphalique : không có hoàn toàn và vĩnh viễn hoạt động não), như thế hạn chế số những người hiến tiềm tàng. Nhưng từ vài năm nay, giới hữu trách y tế của nhiều nước đã cho phép lấy trên những bệnh nhân chết vì một ngừng tim. Ở Pháp, Agence de la biomédecine đã hợp thức hóa nguyên tắc vào năm 2006, nhưng hiện nay điều đó chủ yếu liên quan sự lấy thận và gan.
Khác hẳn những cơ quan này, tim có cái khó đặc biệt là chỉ chịu tình trạng thiếu máu cục bộ trong rất ít thời gian, nghĩa là sự ngừng tuần hoàn máu lấy đi oxy của các tế bào và khiến chúng bị hoại tử. ” Những thí nghiệm của chúng tôi được thực hiện trên động vật rồi trên tim dành cho nghiên cứu đã thuyết phục chúng tôi rằng dẫu sao chúng tôi cũng có một fenetre 30 phút “, BS Stephen Large, người đứng đầu kíp người Anh nhận lấy là đã thực hiện ghép đầu tiên một trái tim “ngừng đập” ở châu Âu.
INCUBATEUR.
Thời hạn ngắn ngủi này buộc rằng người hiến phải chết vì một ngừng tim hô hấp trong môi trường bệnh viện, dưới sự theo dõi của nhân viên y tế. Chính vì vậy những trái tim ngừng đập được ghép phát xuất từ những bệnh nhân nhập viện trong tình trạng chết não, mà những người thân quyết định cho ngừng những điều trị đang được thực hiện vì không còn hy vọng. ” Sự chết được tuyên bố 5 phút sau khi tim ngừng đập. Bắt đầu từ đó, thân thể người chết được đưa vào phòng mổ, ở đây chúng ta cần 6 phút để relancer trái tim “, BS Large đã giải thích như vậy.
Chính ở đó can thiệp prototype được phát triển bởi công ty Hoa Kỳ Transmedics, kết quả của một công trình nghiên cứu 12 năm. Các thầy thuốc ngoại khoa người Úc (St Vincent Hospital, Sydney) và Anh (Papworth, Cambridge) sử dụng nó một cách hơi khác. Những người Úc đặt vào đó tim được lấy để truyền dịch cho đến khi ghép và kiểm tra, trước khi implantation, khả năng co thắt của nó. Còn những người Anh thì trước hết truyền cơ quan mà không lấy nó ra khỏi ngực của người hiến và kiểm tra trong cùng điều kiện khả năng hoạt động của nó. Incubateur Transmedics mà sau đó nó được transvasé cho phép duy trì tim đập trong vài giờ trước khi ghép.
Đối với GS Jean-Noel Fabiani, trưởng khoa ngoại tim mạch thuộc bệnh viện Georges-Pompidou (Paris), đó là một ” cơ hội tuyệt vời để mở rộng số những mẫu ghép”. ” Hiện nay chúng tôi thực hiện 350 trường hợp ghép tim ở Pháp mỗi năm nhưng, khi nhờ đến những tim ngừng đập, ta có thể hy vọng vượt quá mức 500 “, ông đã đánh giá như vậy.
Luật pháp hiện nay ở Pháp không cho phép ghép tim ngừng đập.” Vào thời kỳ văn bản đã được thảo, chúng tôi đã không đệ đơn bởi vì an toàn của bệnh nhân không được đảm bảo “, GS Olivier Bastien , giám đốc lấy và ghép các cơ quan và mổ ở Agence de la biomédecine đã giải thích như vậy. Theo ông, số bổ sung những mẫu ghép có thể thu được như thế ít hơn nhiều 25% được tuyên bố. ” Trên nguyên tắc, điều đó đáng lưu ý, ông công nhận như vậy. Ngoài ra nước Pháp có những kết quả tốt về sự phục hồi chức năng của thận sau một ngừng tim. Nhưng trên một cơ quan nhạy cảm và cốt tử như trái tim, chúng tôi vẫn thận trọng.”
(LE FIGARO 28/3-29/3/2015)
5/ GHÉP CÁC NEURONE CỦA VỎ NÃO : MỘT PREMIERE MONDIALE.
BS Afsaneh Gaillard, trưởng kíp nghiên cứu Inserm ở đại học Poitiers, giải thích một kiểu liệu pháp tế bào mới đã cho phép thay thế những neurone bị thương tổn của vỏ não.
Hỏi : Ở não bộ, vỏ não ở vị trí nào và, tùy theo những vùng của nó, những chức năng khác nhau của vỏ não là gì ?
BS Afsaneh Gaillard : Vỏ não là phần ngoại biên của não, vỏ của nó cũng như lớp ngoài của trái hồ đào (noix). Vỏ não được chia thành nhiều phần, trong đó một chỉ huy, thí dụ vận động (những cử động), một phần khác điều khiển thị giác, một phần khác nữa điều khiển thính giác…
Hỏi : Những thương tổn vỏ não có thường gặp không và nói chung những nguyên nhân của của chúng là gì ?
BS Afsaneh Gaillard : Đó có thể là những bệnh thoái hóa thần kinh, như bệnh Alzheimer, hay những thương tổn do một chấn thương : tai nạn lưu thông, tai biến mạch máu não…
Hỏi : Tùy theo những vùng thương tổn, những phế tật mà những thương tổn này gây nên là gì ?
BS Afsaneh Gaillard : Khi đó là một vùng phụ trách vận động bị thương tổn, có sự mất năng lực thực hiện những cử động : đó là bại liệt, quan trọng ít hay nhiều tùy theo mức độ của thương tổn. Nếu đó là phần điều khiển thị giác bị thương tổn, bệnh nhân có những bất thường thị giác.
Hỏi : Và nếu vùng của vỏ não tương ứng với thính giác bị thương tổn, thương tổn này có gây điếc hay không ?
BS Afsaneh Gaillard : Không, nhưng nó gây nên những vấn đề thính giác bởi vì âm thanh đến não được phân tích ở đó một cách khác : việc xử lý thông tin rất bị rối loạn.
Hỏi : Ngày nay có một điều trị để sửa chữa ở vỏ não những vùng neurone bị thương tổn này hay không ?
BS Afsaneh Gaillard : Bất hạnh thay là không. Còn về những vùng phụ trách vận động, ta kê đơn thực hiện những buổi rééducation với một thầy thuốc kiné liệu pháp. Những kết quả tùy thuộc vào mức độ lan rộng của thương tổn. Nếu mức độ lan rộng giới hạn, những kết quả có thể tốt. Tuổi của bệnh nhân cũng có tầm quan trọng : những thiếu niên phục hồi nhanh hơn những người lớn. Đối với những vấn đề thị giác, một sự phục hồi thị giác có thể cho những kết quả đáng phấn khởi.
Hỏi : Có thể có một quá trình sửa chữa tự nhiên những neurone của vỏ não hay không ?
BS Afsaneh Gaillard : Những tế bào gốc có sửa chữa thương tổn, nhưng chúng không thể thay thế những tế bào bị phá hủy
Hỏi : Định nghĩa của một tế bào gốc là gì ?
BS Afsaneh Gaillard : Một tế bào gốc có đặc điểm là chưa đặc hiệu cho một cơ quan : ta nói là nó “không được biệt hóa” (indifférencié). Nó phát xuất hoặc là từ một phôi thai, hoặc là từ một mô trưởng thành. Những tế bào này, nhờ khả năng biến hóa của chúng (sự biệt hóa), có thể dùng để tái sinh hay tái tạo những mô bị phá hủy : đó là liệu pháp tế bào (thérapie cellulaire)
Hỏi : Mục đích của những công trình của kíp của bà ở Inserm (Poitiers) là gì ?
BS Afsaneh Gaillard : Mục đích là sửa chữa những thương tổn của vỏ não với những neurone lành mạnh, bằng một liệu pháp tế bào cải tiến. Chúng tôi đã sử dụng những tế bào gốc phôi thai có thể biến hóa thành bất cứ loại cơ quan nào.
Hỏi : Bà hãy mô tả cho chúng tôi protocole của công trình nghiên cứu của bà ?
BS Afsaneh Gaillard : Những công trình của chúng tôi được diễn ra thành nhiều giai đoạn.
1. Người ta đã lấy những tế bào gốc phôi thai sau khi thụ thai nhân tạo ở chuột.
2. Người ta cấy chúng trong phòng thí nghiệm để biến hóa chúng nhằm có được những neurone của vỏ não thị giác.
3. Chúng tôi đã gây nên một thương tổn của vỏ não thị giác ở chuột và chúng tôi đã tiêm vào đó những neurone được sản xuất ở phòng thí nghiệm.
Hỏi : Bà đã có được những kết quả nào ?
BS Afsaneh Gaillard : Vùng thương tổn của vỏ não thị giác đã được sửa chữa. Người ta đã tái lập được những circuit não bị thương tổn. Những neurone mới được ghép đã trở nên hoạt động và đã hoàn toàn hội nhập vào não bộ.
Hỏi : Vậy đó là một première mondiale quan trọng.
BS Afsaneh Gaillard : Vâng, đó là lần đầu tiên người ta chứng minh khả năng sửa chữa vỏ não thị giác nhờ những neurone thu được ở phòng thí nghiệm từ những tế bào gốc.
Hỏi : Sự thành công này ở động vật có sẽ dẫn đến sự thực hiện một công trình nghiên cứu ở người ?
BS Afsaneh Gaillard : Hiện nay người ta lên kế hoạch lập lại cùng liệu pháp này nhưng lần này với những tế bào gốc phôi thai người sẽ được ghép ở khỉ. Nếu những kết quả chứng thực khi đó ta sẽ dự kiến một thử nghiệm trên người. Những công trình nghiên cứu của chúng tôi rất đáng phấn khởi nhưng phải thận trọng, những công trình nghiên cứu bổ sung vẫn cần thiết trước khi sử dụng những neurone này trên lâm sàng.
(PARIS MATCH 9/4-15/4/2015)
6/ GHÉP NEURONE ĐỂ SỬA CHỮA NÃO BỘ BỊ HỎNG.
Các nhà nghiên cứu đã có thể thay thế một vùng thương tổn của vỏ não. Một kỳ công được thực hiện ở chuột mở ra những hy vọng mới để điều trị người.
NEUROLOGIE. Sửa chữa vỏ não bị hỏng, cũng như da biết tái sinh để “xóa đi” vết chợt nhỏ nhất. Đó là kỳ công được thực hiện bởi những nhà nghiên cứu Pháp và Bỉ.
Não bộ của chúng ta là một cơ quan mà sức mạnh ngang hàng với sự mỏng manh. Mặc dầu não có một tính dẻo tuyệt vời : đừng cố trở nên contorsionniste ở tuổi trưởng thành, thân thể của bạn sẽ không chịu được, nhưng với vài cố gắng bạn có thể học tiếng tàu hay nghệ thuật quan sát những vì sao. Tuy nhiên, não cũng rất mỏng manh. Nếu một thương tổn xảy ra (do một tai biến mạch máu não, một thương tổn chấn thương, một bệnh thoái hóa thần kinh hay thần kinh…), não không biết tái tạo những tế bào và những nối kết bị biến mất. Giải pháp duy nhất khi đó là học, với cái giá của một sự phục hồi chức năng lâu dài để tìm lại những năng lực bị mất.
” Chúng tôi là những người đầu tiên cho thấy rằng ta có thể sửa chữa vỏ não, trong trường hợp này là vỏ não thị giác “, Alsaneh Gaillard, trưởng khoa nghiên cứu thuộc Laboratoire de neurosciences expérimentales et cliniques (Inserm, université de Poitiers), người điều khiển một kíp nghiên cứu của Inserm và Institut de recherche interdisciplinaire en biologie humaine et moléculaire (Bruxelles) đã giải thích như vậy. Họ đã có thể ghép những neurone trong vỏ não thị giác bị thương tổn của chuột trưởng thành, và quan sát sự tái lập thần kinh cơ thể học và chức năng của vùng não. Một première mondiale đầu tiên được công bố trong tạp chí neuro, và được thực hiện nhờ một vũ khí : liệu pháp tế bào.
CÓ ĐƯỢC NHỮNG NỐI KẾT
Trước hết chúng đã phải có được những neurone tốt phát xuất từ những tế bào gốc phôi thai được cấy in vitro. Khoảng 100 loại neurone khác nhau nằm ở vỏ não (lớp nông của não), được tổ chức thành 6 lớp và thành những vùng phân biệt. Rồi những nhà nghiên cứu đã ghép những tế bào thu được và đã quan sát, trong 12 tháng, hoạt động của chúng. ” Hệ được thiết đặt sau một tháng rưỡi và những nối kết đã được tạo thành, Afsaneh Gaillard lấy làm hớn hở. Chúng tôi đã ghép những progéniteur de neurones visuels, những tế bào chưa thành thục. Những tế bào này sau khi ghép đã còn phát triển, rồi đã thiết lập những nối kết đúng với những neurone thích đáng. Bằng cách kích thích mắt của các con chuột, chúng tôi đã thấy những neurone ghép bị kích hoạt.”
Sau 12 tháng, ghép đã ” bén rễ ” ở 61% các động vật. 6 mẫu ghép trên 47 chứa một tỷ lệ lớn những tế bào không phải neurone, điều này có thể chỉ sự tạo thành một tératome, loại khối u được tạo thành bởi những tế bào không được biệt hóa một cách đúng đắn.
Công trình nghiên cứu gợi ý rằng , để một vỏ não có thể nhận được một ghép neurone, ghép này phải được thực hiện với những neurone tương ứng với vùng bị thương tổn. Như thế, cùng những progéniteur de neurone visuel, được ghép trên một thương tổn vỏ não vận động, đã không cho phép sửa chữa nó ; ngược lại những neurone vận động không hiệu quả trong một vùng thị giác. ” Một trong những thách thức quan trọng của sự sửa chữa của não bộ, các nhà nghiên cứu viết, là tái lập một tính chất nối kết (connectivité) neurone đặc thù và phức tạp. Điều này hàm ý làm phát sinh những mẫu hình đặc biệt các lớp vỏ và vùng não, để tái tạo những mạng cơ năng.”
Afsaneh Gaillard hy vọng từ nay ” có thể có được những loại neurone khác, đặc biệt những neurone vận động. Chúng tôi cũng sẽ trắc nghiệm những ghép này ở khỉ, gần giống với người hơn”. Bởi vì não người phức tạp hơn não của chuột, nhà nghiên cứu đã giải thích như vậy. ” Não người lớn hơn vậy các neurone được ghép sẽ đi xa hơn và phóng những nối kết sợi trục (connexion axonale) lớn hơn.”
(LE FIGARO 13/3/2015)
7/ NHỮNG CÁI ĐƯỢC THUA QUAN TRỌNG CỦA NGOẠI KHOA NGÀY MAI ?
Professeur Philippe Cinquin
Professeur à l’université Joseph-Fourier
Praticien hospitalier au CHU de Grenoble
Mổ một bệnh nhân với mức độ chính xác hơn và ít nguy cơ hơn, đạt đến một cơ quan nằm sâu trong bụng nhờ một đường xẻ chỉ vài milimet, tạo một hình ảnh ảo 3 chiều không gian (image virtuelle 3D) của mô mà ta phải điều trị..Những kỳ công này và những kỳ công khác từ nay có thể được thực hiện nhờ ngoại khoa được hỗ trợ bởi máy vi tính (chirurgie assistée par ordinateur), lãnh vực đang phát triển mạnh từ 15 năm nay.
Những “geste médico-chirurgical” (những động tác nội-ngoại khoa) được hỗ trợ bởi máy vi tính (GMCAO : gestes médico-chirurgicaux assistés par ordinateur) đã xuất hiện ở Grenoble vào cuối những năm 1980 với sự robot hóa (robotisation) của stéréotaxie, một kỹ thuật được sử dụng trong ngoại thần kinh để đạt đến những vùng của não bộ một cách rất chính xác. Dần dần, GMCAO được mở rộng đến những lãnh vực lâm sàng khác như chỉnh hình, urologie.. Những champ des possible trong lãnh vực này vẫn còn lớn.
Đó là mục tiêu của chúng tôi khi chúng tôi thành lập Cami (Computer Assisted Medical Interventions), phát sinh từ sự phối hợp của 6 phòng thí nghiệm dưới sự chỉ đạo của Inserm, của CNRS, của institut Telecom và các đại học. Tất cả chúng tôi chia xẻ ý tưởng rằng GMCAO đã chưa phát triển hết tiềm năng của mình. Chúng tôi đã quyết định hợp nhất những kiến thức của chúng tôi và những năng lực chuyên môn của chúng tôi để tạo dựng ngoại khoa của ngày mai và cải thiện những kết quả của mọi can thiệp ngoại khoa bằng cách dựa trên những kỹ thuật mới nhất của ngoại khoa được hỗ trợ bởi máy vi tính (chirurgie assistée par l’ordinateur). Chúng tôi làm việc chặt chẽ với những kíp lâm sàng và công nghiệp.
Phòng thí nghiệm Cami này, mà chúng tôi đã thành lập hồi tháng 9, có mục tiêu làm dễ những dự án hợp tác. Đó không phải là những trao đổi giai đoạn giữa các thầy thuốc ngoại khoa nêu ra một vấn đề và những nhà nghiên cứu phát minh một giải pháp. Một sự cộng tác thật sự trên cơ sở dài lâu được xác lập : từ sự quan sát chăm chú những giới hạn được gặp trong thực hành của những thầy thuốc ngoại khoa giỏi nhất nảy sinh sự nhận diện những vấn đề khoa học đôi khi phức tạp. Những câu trả lời độc đáo có thể được đề nghị, nhờ tính chất rất đa khoa của những năng lực được tập hợp trong Cami (toán học áp dụng, informatique, xử lý tín hiệu, robotique, biomécanique, khoa học giáo dục..). Những giải đáp này được xây dựng trong những luận án hay những công trình nghiên cứu cho phép nghĩ ra những démonstrateur, nhờ đó những những preuve de concept có thể có được và được trắc nghiệm trên lâm sàng.
Những cái được thua có nhiều đối với thầy thuốc ngoại khoa : ” thấy xa hơn cái có thể nhìn thấy” (voir au-delà du visible), được hổ trợ trong việc lấy những quyết định phức tạp trong một môi trường không chắc chắn, rồi trong sự thực hiện chiến lược tối ưu, có thể tự đào tạo trên những stimulateur réaliste và chứng minh lợi ích lâm sàng của những cải tiến được đề nghị. Trong Cami, chúng tôi phát minh cả một loạt những giải đáp cho những thách thức này : những phương tiện chụp hình ảnh cải tiến có khả năng thấy không những hình dáng mà còn cả chức năng của các mô của phẫu trường, những phương pháp “réalité augmentée” trình bày đồng thời cho nhà ngoại khoa hình ảnh của những caméra cổ điển và kết quả của sự xử lý nhiều capteur, những phương pháp phân tích tự động chất lượng của động tác được thực hiện, những robot miniaturisé có khả năng hợp tác với người phẫu thuật…
(LE FIGARO 6/1/2015)
8/ SỐ NHỮNG TRƯỜNG HỢP MỔ RUỘT THỪA VIÊM SỤT GIẢM RÕ RỆT Ở PHÁP.
Tiến bộ của chụp hình ảnh đã cho phép làm giảm những can thiệp vô ích.
CHIRURGIE. Một điểm đau ở phía phải của bụng và mối nghi ngờ nảy sinh tức thời : phải chăng đó là ruột thừa viêm ? Thật vậy, bệnh lý này thường gặp : cứ 15 người thì có một bị bệnh này, chủ yếu giữa 10 và 30 tuổi. Nếu những tiến bộ y khoa đã cho phép chia 4 số những trường hợp mổ trong 30 năm qua, như được chứng tỏ bởi một báo cáo của Drees (direction des statistiques của Bộ y tế), được công bố đầu tháng hai, sự tranh luận về sự cải thiện của những thực hành y khoa vẫn còn mở ngõ.
Nhờ những tiến bộ y khoa (chụp cắt lớp vi tính, siêu âm) và những phân tích sinh học đã cho phép làm tinh tế hơn chẩn đoán, sự sụt giảm những can thiệp ngoại khoa trong những năm qua là ngoạn mục, công trình nghiên cứu đã tiết lộ như vậy. Giữa năm 1986 và 2012, sự thụt lùi đã diễn ra hằng định, chuyển từ một tổng cộng 300.000 trường hợp mới mỗi năm vào năm 1986 xuống còn 83.000 năm 2012, hoặc một sự thụt lùi 8% mỗi năm. Sự sụt giảm nhanh chóng này đã cho phép nước Pháp, vốn luôn luôn mổ nhiều ruột thừa viêm hơn những nước khác, xích lại gần chuẩn mực. Năm 2009, số những trường hợp mổ 1,4 lần nhiều hơn so với Hoa Kỳ và 1,7 lần nhiều hơn Vương Quốc Anh (so với 2,5 và 3,7 lần vào năm 1993).
Ngoài ra, công trình nghiên cứu phát hiện một tiến triển hai tốc độ, tùy theo tính chất của ruột thừa viêm mà bệnh nhân có. Ruột thừa viêm không biến chứng được gọi là ” đơn giản”, thuộc loại can thiệp thụt lùi mạnh, nhất là ở trẻ em, trong khi đó những dạng trầm trọng, bao gồm viêm phúc mạc, liên quan thường hơn hai thái cực của đời sống, vẫn trong tình trạng ổn định.
Mặc dầu những tiến bộ được chứng thực, những cuộc tranh luận vẫn gay gắt giữa những chuyên gia về những phương tiện để làm giảm hơn nữa những can thiệp vô ích mà không mang lại nguy cơ đối với các bệnh nhân. Một buổi hội thảo của Académie de chirurgie vào tháng năm 2012 đã mở ra một cuộc tranh luận với một thời kỳ suy nghĩ 2 năm tiếp theo sau để hiệu chính những khuyến nghị chính thức.
Theo BS Philippe Montupet, chủ tọa nhóm công tác dành cho dự án này, trước hết đã nhắc lại rằng trong trường hợp nghi ngờ về chẩn đoán nhà ngoại khoa sẽ không bị khiển trách nếu đã lấy đi một ruột thừa của một bệnh nhân. ” Vẫn luôn luôn tốt hơn là tránh có thể mang lại những biến chứng “, ông đã giải thích như vậy. Cũng vậy, mặc dầu những ưu điểm của nội soi (mổ qua một đường xẻ bé nhỏ) được công nhận, đặc biệt nó hạn chế vài biến chứng hậu phẫu, nhưng phương pháp này đòi hỏi một đào tạo ngoại khoa đặc biệt không hề bị bắt buộc. Một cách tổng quát hơn, mọi thể “biến chứng” cần phải được mổ ngay.
Điều đó đặt ra vấn đề chẩn đoán, phải là lãnh vực của thầy thuốc ngoại khoa, BS Montupet đã xác nhận như vậy. ” Không nên tầm thường hóa những nguy cơ của ruột thừa viêm. Càng ngày ta càng thấy những trường hợp biến chứng đến bệnh viện chúng ta bởi vì chẩn đoán đã bị trì hoãn”, ông đã phàn nàn như vậy. Tỷ lệ tử vong do ruột thừa viêm vẫn thấp và ổn định, nhưng tỷ lệ này có thật, khoảng 0,02%.
Còn lại vấn đề gai góc về lợi ích điều trị của các kháng sinh. Vấn đề này đã là đối tượng của những công trình nghiên cứu để xác định chúng có thể, trong vài trường hợp, tránh khỏi bị mổ hay không. Ở Pháp việc kê đơn những kháng sinh lúc bị ruột thừa viêm đơn thuần để trì hoãn cuộc mổ vài giờ đã trở nên thông thường. Nhưng giả thuyết được nghiên cứu bởi GS Corinne Vons, trưởng khoa chirurgie gastrique của CHU Jean-Verdier, Bondy (Seine-Saint-Denis), nhằm cho kháng sinh trong một thời gian để loại bỏ nhiễm trùng cần phải được xác định.
Những công trình đầu tiên, được công bố năm 2011 trong tạp chí khoa học uy tín The Lancet, đã gây nên ở Pháp một cuộc tranh luận sôi nổi về chủ đề này. ” Khi tôi bắt đầu những công trình này, chúng đã gây nên một mối lo sợ thật sự, GS Vons đã báo cáo như vậy. Nhưng hôm nay, tư duy đã tiến triển. Điều đó không còn được xem là một sự điên rồ.” Tuy nhiên hướng này vẫn còn ở giai đoạn thí nghiệm và không được đề cập như một phương pháp điều trị trong những khuyến nghị mới nhất của Bộ Y Tế năm 2012.
(LE FIGARO 21/2/2014)
9/ LASER VASCULAIRE DÙNG TRONG MỤC ĐÍCH GÌ ?
Docteur Jean-Michel Mazer
Directeur médical du Centre international
de la peau de Paris
Vice-président du Groupe laser de la Société
française de dermatologie.
Chúng ta sử dụng laser trong dermatologie từ 30 năm nay và những laser vasculaire đã nằm trong số những laser đầu tiên được sử dụng. ta hiểu ” laser vasculaire” là một laser có khả năng gây nên sự xơ cứng, nghĩa là sự co lại của các mạch máu bi giãn quá mức. Ở da, điều đó tương ứng với nhiều trường hợp bệnh lý, như couperose (bệnh sùi đỏ mặt), những giãn tĩnh mạch (varicosités) của các cẳng chân hay angiome plan (u mạch phẳng), thường được gọi không đúng là những “tache de vin”. Ít được biết hơn, những radiodermite mãn tính xuất hiện dưới dạng những placard, thường có dạng hình học, được cấu tạo bởi nhiều mạch máu bị giãn, do bức xạ khi điều trị ung thư, thường là một ung thư vú ; như thế vùng trước ức thường bị thương tổn, bất hạnh thay nhắc lại cho bệnh nhân bệnh ung thư của mình. Trong tất cả các trường hợp này, các laser vasculaire hôm nay tạo nên một giải pháp điều trị đặc biệt đáng lưu ý, đã thay đổi tiến triển của những bệnh lý khác nhau này.
LASER VASCULAIRE LÀ GÌ ?
Một laser là một nguồn năng lượng phát ra một ánh sáng có những tính chất đặc biệt, trong đó có một độ dài sóng đặc thuc. Những photon tạo nên ánh sáng này sẽ được hấp thụ một cách đặc hiệu bởi cible élective của những laser vasculaire : đó là hémoglobine hiện diện bên trong những hồng cầu. Như thế, nhung huyết quản bị giãn là một mục tiêu chọn lọc. Ánh sáng được hấp thụ sẽ đuoc biến đổi thành nhiệt lượng, gây nên một tác dụng nhiệt lên thành của huyết quản, điều này sẽ gây nên sclérose, nghĩa là sự co lại dần dần trong những tuần tiếp theo.
Trên phương diện lịch sử, những laser huyết quản đầu tiên đã được sử dụng để điều trị những angiome plan. Những kết quả có thể thấy được, nhưng những laser này có nguy cơ gây sẹo. Kíp của Rox Anderson (Boston) đã mang lại một tiến bộ rất quan trọng khi chứng tỏ rằng nếu sử dụng những thời gian cực ngắn của những xung động có thể vẫn rất đặc hiệu và tránh mọi tác dụng nhiệt ra ngoài những huyết quản này. Điều này đã tối ưu tolérance, bằng cách tránh nguy cơ sẹo, và đã cho phép điều trị những angiome plan ngay giai đoạn còn non nhất. Trên thực hành, điều trị có thể được bắt đầu một cách an toàn ngay những tuần lễ đầu tiên. Cơ chế tác dụng đặc biệt này dựa trên một hiệu quả làm đông các mạch máu, được thể hiện bởi điều mà chúng tôi gọi là purpura nhưng tương ứng đối với bệnh nhân một dạng vẻ echhymose. Điều này tồn tại trong một đến hai tuần , nhưng là biểu hiện không phải một vết bỏng mà là một hiệu quả của coagulation. Có thể tránh điều này bằng cách sử dụng những paramètre khác nhau, nhưng trong trường hợp này tính hiệu quả sẽ ít tốt hơn trên những angiome và điều này phai được dành cho điều trị couperose.
NHỮNG CHỈ ĐỊNH CHÍNH CỦA LASER VASCULAIRE
Trước hết là couperose. Điều mà chúng ta thường gọi là couperose cũng được gọi là “rosacée”. Đó là một bệnh rất thường gặp, được biểu hiện bởi những chấm đỏ trên da (rougeur) của má, mũi, đôi khi cằm. Những chấm đỏ này thường gia tăng nhiều khi thay đổi khí hậu hay cảm xúc. Chúng biểu hiện một sự giãn các huyết quản của bì ở những vùng nằm ở trung tâm của mặt. mặc dầu những điều trị mỹ phẩm, và sự bảo vệ chống lại ánh sáng mặt trời là hữu ích để bảo vệ chất lượng của đa, tia laser đã hoàn toàn biến đổi tiến triển của căn bệnh này. Hai laser vasculaire chính là những laser KTP và những laser à colorant pulsé. Điều trị đã được thực hiện bởi một thầy thuốc được đào tạo theo kỹ thuật này. Hai phương pháp được đề nghị : phương pháp được sử dụng để điều trị các angiome, dựa trên sự làm đông các huyết quản, và phương pháp nhằm tránh aspect affichant này đối với bệnh nhân. Khi những thương tổn huyết quản là rất mạnh, có tính chất ” érythrosique “, nghĩa là được đánh dấu bởi một màu hồng lan tỏa, khi đó phương pháp bắt chịu ban xuất huyết (purpura) (nghĩa là aspect affichant và khó hóa trang trong vài ngày) là hiệu quả hơn phương pháp cho phép tránh những vết tạm thời này. Bệnh nhân phải được báo cho biết về điều đó : nếu bệnh nhân muốn tránh những vết bầm tím, bệnh nhân có thể thử phương pháp thứ hai nhưng nói chung sẽ cần nhiều séance hơn. Khi đó, đó là sự lựa chọn của bệnh nhân, phương pháp thứ hai có những di chứng đơn giản hơn. Ngược lại, nếu bệnh nhân có một couperose được đánh dấu chủ yếu bởi những huyết quản giãn nở rõ rệt, có thể thấy được bằng mắt trần, thì kỹ thuật thứ hai, đơn giản hơn trong thực hành, khi đó cũng hiệu quả. Vậy quyết định này phải là kết quả của một cuộc tranh luận được soi sáng giữa bệnh nhân và thầy thuốc thực hành. Nói chung, số những buổi điều trị có thể là hai hoặc ba đối với một couperose trung bình ; đôi khi con số này có thể vượt quá. Sự chênh lệch giữa hai buổi phải là khoảng hai tháng. Những kết quả tương đối dài lâu. Nhất là khi kết quả sau cùng là tốt nhất. Như thế, thường ưa thích hơn làm thêm một séance ngay từ đầu, để tối ưu hóa kết quả, khi đó tính hiệu quả kéo dài gần 10 năm.
Trong vài trường hợp, couperose được kèm theo bởi những thương tổn viêm thường được gọi là acné rosacée. Nguồn gốc của thương tổn viêm này không được xác định rõ. Hiện nay người ta quy vai trò của một vi khuẩn nhỏ hiện diện trong một ký sinh trùng sống tự nhiên ở da. Cuối cùng, một sự cải thiện rõ rệt của những thương tổn huyết quản bởi laser không luôn cho phép phong bể những thương tổn này. Ta quan sát rằng, mặc dầu những điều trị kháng viêm nói chung đuoc đảm trách bởi Sécurité Sociale, không hề là như vậy đối với điều trị laser.
Thứ hai, những angiome plan. Chúng là chỉ định y khoa quan trọng của những laser vasculaire ngay khi đứa bé sinh ra đời. Lợi ích của điều trị sớm là hiển nhiên : tránh ảnh hưởng tâm lý của những thương tổn này, nói chung không chịu được, đặc biệt khi đứa bé bắt đầu đi học. Trong vài trường hợp, chúng chịu trách nhiệm sự thất bại học hành, sự hội nhập xã hội kém và ta hiểu rõ sự khó khăn khi phải sống với một angiome plan trên mặt trong xã hội của chúng ta, là nơi bề ngoài có một tầm quan trọng thường quá lớn. Điều trị ở trẻ em phải được thực hiện với một laser à colorant pulsé. Ở người trưởng thành ta có thể sử dụng không phân biệt laser à colorant pulsé và laser KTP. Những buổi laser được lập lại mỗi 3 đến 4 tháng và bệnh nhân phải được đả thông rằng số lượng các buổi điều trị nói chung là quan trọng. Bất hạnh thay, với thời gian nhìn lại ngày nay ta không thể nói rằng ta có thể xóa bỏ hoàn toàn những thương tổn này. Trong vài trường hợp, điều đó có thể thực hiện, nhưng trong khoảng 1/4 các trường hợp, những thương tổn là dai dẳng và, mặc dầu những tiến bộ có thể thấy được và rõ rệt, chúng không hoàn toàn thỏa mãn. Những tiến bộ mới đây hướng về sự kết hợp những điều trị bằng laser với sự kê đơn những thuốc có thể ngăn cản sự giãn của các huyết quản, đôi khi do điều trị. Thật vậy, khi ta thực hiện một điều trị bằng laser, laser sẽ gây nên sự xơ cứng của những huyết quản, nhưng, trong những tuần tiếp theo sau, ta quan sát sự tạo thành những huyết quản mới. Như thế kết quả thật sự của một buổi laser là sự khác biệt giữa điều đã được làm xơ cứng và điều đã xuất hiện trước buổi laser kế tiếp.
Nhiều công trình nghiên cứu với những loại thuốc khác nhau hiện đang được thực hiện. Điều trị những angiome plan được Sécurité sociale đảm nhận từ năm 1994.
NHỮNG CHỈ ĐỊNH KHÁC
Về những varicosité của cẳng chân, điều trị ưu tiên vẫn là tiêm chất làm xơ cứng (injection sclérosante) mà chúng ta biết rõ. Tuy nhiên, khi các huyết quản trở nên rất mảnh, chằng chịt với nhau, thì trên thực hành không thể làm xơ cứng những huyết quản này bằng kim. Khi đó những laser có thể được đề nghị bổ sung cho những mũi tiêm làm xơ cứng. Khi những huyết quản hơi giãn nở thêm, thì các laser có thể là một giải pháp thay thế tránh kim tiêm.
Ta nhấn mạnh lợi ích của laser à colorant pulsé trong điều trị những radiodermite chronique sau radiothérapie. Nhờ những kỹ thuật này và nhờ tiến bộ của những hóa trị và ngoại khoa, tiên lượng của nhiều ung thư, đặc biệt của vú, đã được cải thiện nhiều. Tuy nhiên, đặc biệt đối với ung thư vú, vài phụ nữ sau radiothérapie có những mảng (placard) dạng hình học, đỏ, bởi vì được tạo thành bởi nhiều huyết quản. Những công trrinh nghiên cứu đã cho thấy rằng hai hay bốn buổi laser à colorant pulsé cho phép có được những kết quả rất tốt mà không bị nguy cơ, bằng cách sử dụng những paramètre giống với những paramètre được sử dụng khi ta điều trị những angiome plan ở những trẻ sơ sinh. Những laser vasculaire đôi khi được đề nghị trong nhiều bệnh, những điều này đặc biệt gây tranh cãi, đặc biệt do thiếu công trình nghiên cứu. Tương lai sẽ cho phép định rõ hơn những điểm này. Trong vài loại angiome, như angiome tubéreux, khác hẳn những angiome plan (chúng khác nhau bởi khuynh hướng tăng thể tích trong những tuần lễ đầu của đời sống, nhưng cũng bởi sự thoái biến thứ phát sau vài nam) laser không được chỉ định. Trong những trường hợp ngoại lệ, đặc biệt khi những thương tổn bị loét ở trẻ sơ sinh, laser có thể được đề nghị cấp cứu. Một cách tổng quát, không có vai trò, nhất là điều trị đã được cách mạng hóa gọi sự khám phá của một équipe de dermatologie của CHU de Bordeaux, đã chứng tỏ lợi ích của các betabloquant.
Kết luận, những laser vasculaire được sử dụng rộng rãi ở dermatologie. Chỉ định chủ yếu của chúng là những angiome plan, bất cứ lứa tuổi nào. Chúng đã chứng tỏ tính hiệu quả và vô hại. Điều trị couperose cũng hưởng lợi nhiều nhờ laser vasculaire. Chúng cho phép gây nên sự thoái biến của những thương tổn này. Chúng phải được thực hiện bởi những thầy thuốc được đào tạo theo kỹ thuật này, điều trị các angiome plan vẫn là lãnh vực của các thầy thuốc chuyên về bệnh ngoài da.
(LE FIGARO 23/9/2013)
Ghi chú :
– Couperose : sự giãn thường trực và có thể trông thấy của những huyết quản nhỏ của da mặt. Couperose là một bệnh rất thường gặp , chủ yếu xảy ra ở các phụ nữ có da sáng và mảnh, giữa 30 và 50 tuổi. Cũng có thể gặp ở đàn ông. Couperose được thể hiện bởi những giận mạch máu ngoại biên nhỏ (télangiectasie), tạo nên những đường mảnh màu đỏ hay tím, hình cành cây, nằm đối xứng trên mũi và các gò má. Điều trị duy nhất có hiệu quả là phá hủy bằng dao điện hay laser vasculaire.
– Angiome capillaire : “Tache de vin” hiện diện lúc sinh và tồn tại sau đó. Trên đầu nó có thể được liên kết với angiome méningé là một nguyên nhân hiếm của động kinh.
10/ TA CÓ THỂ MỔ CÁC VARICES CỦA CÁC CHI DƯỚI MÀ KHÔNG GÂY SẸO MỔ KHÔNG ?
Professeur Olivier Goeau-Brissonière
Président de la Société de chirurgie vasculaire
Centre hospitalier universitaire Ambroise-Paré.

Những giãn tĩnh mạch (varices) của các chi dưới (những tĩnh mạch ngoại biên bị giãn và là nơi của một hồi lưu về phía các chân) là một trong những bệnh lý thuờng gặp nhất của những người trưởng thành, vì lẽ 6-8 phụ nữ trên 10, và 3 đến 5 người đàn ông trên 10 bị ở những mức độ khác nhau trong suốt đời họ và 25% sẽ cần những điều trị nội khoa và ngoại khoa. Chúng cũng thường xảy ra ở những tĩnh mạch hiển (veines saphènes) lớn và nhỏ. Hai tĩnh mạch này là những tĩnh mạch chính của hệ tĩnh mạch ngoại biên. Hệ tĩnh mạch ngoại biên chỉ chiếm 10% những tĩnh mạch đưa máu về tim, phần còn lại của hồi lưu tĩnh mạch (retour veineux) được đảm bảo bởi những tĩnh mạch sâu.
Đó là một bệnh mãn tính có tiến triển chậm được làm dễ bởi lối sống của các nước công nghiệp. Ta phân biệt tùy theo dạng vẻ của chúng và tùy theo kích thước của chúng những télangiectasies, những varices réticulées và những varices có đường kính trên 3 mm. Chúng có thể đặt ra những vấn đề thẩm mỹ nhưng cũng là nguyên nhân của những triệu chứng khác nhau (nặng hay đau cẳng chân, phù, ngứa, crampe vào ban đêm, bồn chồn), thậm chí những biến chứng nhu viêm hay hypodermite, paraphlébite hay huyết khối tĩnh mạch nông (thrombose veineuse superficielle), xuất huyết giãn tĩnh mạch (hémorragie variqueuse), loét cẳng chân. Sự phòng ngừa và điều trị những thể không biến chứng phải được ưu tiên hơn là chờ đợi những thương tổn nặng hơn.
MỘT ECHO-DOPPLER LÀ CẦN THIẾT
Vài yếu tố tạo thuận không thể kiểm soát được : tuổi tác, nữ giới, những điều trị hormone, thai nghén. Những yếu tố có thể kiểm soát và như thể ngăn ngừa sự xuất hiện hay sự trầm trọng của các varices: sự thặng dư thể trọng, tư thế đứng, piétinement professionnel, những rối loạn statique plantaire, sự tiếp xúc với nhiệt (sauna, épilation, bain…), tình trạng không hoạt động vật lý.
Những lý do khám bệnh khác nhau : thẩm mỹ, chức năng trong trường hợp triệu chứng, trong trường hợp biến chứng. Trong tất cả các trường hợp, một écho-doppler là cần thiết. Nó cho phép xác nhận chẩn đoán, định rõ mức cơ thể học và mức độ thương tổn. Nó cho phép hướng định sự lựa chọn điều trị.
Sclérothérapie mà nguyên tắc là tiêm vào trong tĩnh mạch giãn một chất kích thích gây co thắt tĩnh mạch rồi khiến thành bị dày lên và nhanh chóng làm bít tĩnh mạch : Những bất tiện chủ yếu là thời gian điều trị (nhiều buổi) và tái phát. Hiện nay tiêm chất làm xơ cứng (sclérosant) dưới dạng mousse làm gia tăng tính hiệu quả của sclérothérapie. Sclérothérapie cần một guidage échographique. Tỷ lệ tái phát khoảng 27% lúc 5 năm, những con số giống với những con số của crossectomie-stripping, nhưng tốt hơn trong trường hợp những kỹ thuật nội tĩnh mạch phá hủy bằng nhiệt (technique endoveineuse de destruction thermique). Những kỹ thuật này được thực hiện lúc khám ở phòng mạch. Những biến chứng là hiếm và thường hiền tính. Không có sẹo nhưng có thể có sự nhuộm màu của da (thường tạm thời) đó phản ứng của thuốc gây xơ cứng.
Về phlébectomie (cắt bỏ tĩnh mạch), can thiệp nhằm hủy bỏ những tĩnh mạch giãn (veines variqueuses) bằng những đường xẻ nhỏ nhiều tầng (incisions étagées), nó gần như không để lại dấu vết. Riêng rẻ, chúng có thể được thực hiện ngoai trú với gây tê tại chỗ và chủ yếu có mục đích thẩm mỹ.
Crossectomie-stripping nhằm cắt bỏ tĩnh mạch hiển (veine saphène) bằng phẫu thuật, bởi hai đường xẻ ngắn. Can thiệp này thường được liên kết với những phlébectomie. Đó là can thiệp quy chiếu, chủ yếu trong trường hợp những varices quan trọng. Những can thiệp này đã được tiêu chuẩn hóa và được nắm vững bởi các thầy thuốc chuyên khoa mạch máu. Nó có thể được thực hiện với gây tê tại chỗ, loco-régionale hay tổng quát và trong phần lớn các trường hợp có thể được thực hiện ngoại trú. Những tác dụng phụ sau mổ thường là nhẹ (đau, bầm tím).
NHỮNG KỸ THUẬT NỘI TĨNH MẠCH
Đối với những kỹ thuật nội tĩnh mạch (technique endoveineuse), dầu đó là tần số phóng xạ (radiofréquence), laser nội tĩnh mạch hay hơi nước nóng (vapeur d’eau chaude), mục đích của chúng là phát ra từ bên trong tĩnh mạch một năng lượng gây nên một sự phá hủy nhiệt (destruction thermique) với phản ứng sợi (réaction fibreuse) và sự co lại của thành mạch không còn để lại đường lạch (chenal) cho sự tuần hoàn máu, nhiên hậu dẫn đến sự biến mất của tĩnh mạch được điều trị. Chỉ có những tĩnh mạch hiển là tiếp cận được với loại điều trị này. Những giãn tĩnh mạch nằm ở những nhánh bên được điều trị đồng thời hay sau đó bởi phlébectomie hay sclérose.
Những kỹ thuật này được thực hiện ngoại trú với gây tê tại chỗ, và cho phép một sự tái tục hoạt động bình thường, nghề nghiệp và thể thao. So với crossectomie-stripping, chúng không gây sẹo ở nếp bẹn hay sau đầu gối, cũng không cần lấy đi thân chính của những tĩnh mạch hiển. Chúng phải được thực hiện trong những điều kiện vô trùng tốt và dưới sự kiểm soát của siêu âm. Những biến chứng thường nhẹ (phản ứng viêm, những bầm máu) và nguy cơ tái phát dưới 10% lúc 2 năm. Kỹ thuật hơi nước nóng (technique à la vapeur) đang được đánh giá.
Không nên quên những điều trị phòng ngừa và liên kết. Vì điều trị nội khoa cho phép làm giảm những triệu chứng, nhưng không cho phép dập tắt tiến triển. Nhưng việc mang những bas de compression trong những tình huống nguy cơ (piétinement, position statique, voyage aérien…) rất được khuyến nghị. Điều trị này gây gò bó nhưng rất hiệu quả trong trường hợp có triệu chứng và biến chứng. Trái lại compression bị chống chỉ định trong trường hợp bệnh động mạch nghiêm trọng của các chi dưới.
Kết luận, không có điều trị duy nhất và vĩnh viễn của các varices. Sự sử dụng kỹ thuật này hay kỹ thuật kia tùy thuộc vào tầm quan trọng và mức độ lan rộng của các varices, đường kính của thân tĩnh mạch hiển, corpulence của các bệnh nhân, tuổi tác và hoạt động của họ trong cuộc sống thường ngày và nghề nghiệp. Sự lựa chọn tốt sẽ được đề nghị bởi các thầy thuốc mạch máu có toàn bộ những phương tiện điều trị. Trong tất cả các trường hợp, một theo dõi đều đặn và lâu dài được khuyến nghị.
Ghi chú :
– Télangiectasie : sự giãn thường trực của một tĩnh mạch nhỏ (tiểu động mạch, mao mạch, tiểu tĩnh mạch) nằm trong bì (derme).
– Stripping : kỹ thuật saphénectomie (cắt bỏ tĩnh mạch hiển), được thực hiện trong trường hợp bất túc tĩnh mạch chi dưới.
(LE FIGARO 2/7/2012)
BS NGUYỄN VĂN THỊNH
(27/04/2015)

Thời sự y học số 369 – BS Nguyễn Văn Thịnh

1/ SỰ PHÁT TRIỂN TĂNG VỌT CỦA QUANG TUYẾN CAN THIỆPtsyh369Từ những năm 2000, bùng nổ những động tác điều trị (actes thérapeutiques) đi vào trường hoạt động của các thầy thuốc X quang.
Quang tuyến can thiệp (radiologie interventionnelle) hay quang tuyến điều trị (radiologie thérapeutique) cho phép đạt rất chính xác một đích điều trị (cible thérapeutique), dưới sự kiểm soat của siêu âm hay scanner hay một thăm dò chụp hình ảnh khác hay những đường tự nhiên, xuyên qua da, nhờ một ống thông rất mảnh.
“Để điều trị, các thầy thuốc X quang có thể sử dụng những kỹ thuật khác nhau : emboliser (bít) một mạch máu xuất huyết hay phải tiêu hủy nó bởi vì nó nuôi dưỡng một khối u, đưa một thiết bị nhỏ vào trong một động mạch (stent) để tránh không cho nó bị bít thêm nữa, phá hủy một nodule bằng lạnh (cryothérapie) hay bằng đốt (micro-onde, radiofréquence). Những khả năng khác : điều trị bằng hóa trị (chimiothérapie) một cách trực tiếp trong lòng khối u hay bằng những bille radioactive… Khả năng lựa chọn là rộng”, BS Audrey Fohlen, thuộc khoa chụp hình ảnh chẩn đoán và quang tuyến điều trị của CHU de Caen, đã giải thích như vậy.
Vì phần lớn các động tác rất là kỹ thuật, các thầy thuốc X quang thực hiện chúng đã chuyên môn hóa. “Trong lãnh vực mạch máu thần kinh (neuro-vasculaire), sự làm tắc nghẽn các phình mạch nội sọ (embolisation des anévrismes intracraniens), hoặc trước khi chúng bị vỡ (cấp cứu), đã gần như thay thế ngoại thần kinh trong chỉ định này, GS Alain Bonafé, coordinateur của khoa chẩn đoán thần kinh (département de neurodiagnostic) thuộc CHU de Montpellier đã ghi nhận như vậy.Ngoài việc điều trị những dị dạng huyết quản (malformation vasculaire), chúng tôi cũng ở tuyến đầu để làm phân rã cơ học (désintégration mécanique) những cục máu đông lớn bít những huyết quản của não. Một công trình mới đây xác nhận lợi ích của nó so với đường tĩnh mạch. Hy vọng là hạn chế hơn nữa những di chứng sau một tai biến mạch máu não bằng cách mở lại càng nhanh càng tốt động mạch bị bít. Tuy nhiên chúng tôi phải nhận rất nhanh những bệnh nhân liên hệ, trong những giờ đầu tiên hết sau tai biến mạch máu não. Đó không phải là trường hợp hiện nay, do đó nguy cơ phân biệt giữa những bệnh nhân sẽ có thể nhận kỹ thuật này với những bệnh nhân khác.
Can thiệp trên những xuất huyết tại chỗ ngoài não cũng thuộc vào việc thường tình : ” Trong phụ khoa, thầy thuốc X quang là trung tâm của điều trị hémorragie de la délivrance và của điều trị những fibrome, bằng cách can thiệp bằng embolisation”, GS Jena-Pierre Pelage, trưởng khoa quang tuyến của CHU de Caen, đã xác nhận như vậy. Trong lãnh vực xuất huyết dạ dày ruột, các thầy thuốc X quang thực hiện những embolisation đối với những giãn tĩnh mạch thực quản hay những động mạch của dạ dày hay của tá tràng, chịu trách nhiệm những xuất huyết tiêu hóa. Đó không phải là lãnh vực hoạt động duy nhất của họ. ” Cũng có thể điều trị những u gan, kể cả khi đó là những u thứ phát một ung thư khác (những di căn) “, BS Fohlen đã nhấn mạnh như vậy. Và nguyên tắc cũng tương tự trong phổi học : ” Những thầy thuốc X quang có thể can thiệp cấp cứu trên một động mạch phế quản hay phổi xuất huyết ở những người ho ra máu. Cũng có thể khử bỏ một u phổi bằng cách tiêu hủy nó bằng nhiệt (radiofréquence).Khi không thể mổ được bởi vì người bệnh không chịu được nó, đó là một giải pháp mang lại những cơ may thành công tốt, nhất là đối với những khối u dưới 3 cm”, GS Pelage đã nhấn mạnh như vậy.
Niệu học (urologie) và thận học (néphrologie) cũng không thua kém gì : ” Các thầy thuốc X quang có thể đốt, bằng tần số phóng xạ, những u nhỏ dưới 4 cm của thận, thường không có triệu chứng và được khám phá tình cờ nhân một thăm dò chụp hình ảnh được đòi hỏi vì một nguyên nhân khác. Thường đó là những u ung thư mà một vài nhiên hậu lớn lên và trở nên nguy hiểm. Thế mà, được điều trị ở giai đoạn này, các kết quả có được là tốt (không nhiều tái phát hơn so với một cắt bỏ thận bán phần) và đòi hỏi dưới 24 giờ nhập viện”, GS Patrice Taourel (CHU Montpellier) đã xác nhận như vậy.
Một lãnh vực khác trong đó radiologie thérapeutique rất được áp dụng là lãnh vực xương và khớp, như GS Catherine Cyteval (CHU Montpellier) đã giải thích như vậy. ” Những đau xương khớp là một triệu chúng thường gặp. Nhiều khả năng cho các thầy thuốc X quang trong trường hợp thất bại những thuốc giảm đau thông thường, từ tiêm ngấm (infiltration) các thuốc kháng viêm dưới sự kiểm soát của chụp hình ảnh để tiêm chất ciment để làm cứng một gãy đốt sống gây đau. Sự phá hủy những u xương nhỏ cũng có thể được thực hiện, bằng radiofréquence, cryothérapie hay bằng làm xơ cứng (scléroser) những mạch máu nuôi dưỡng thương tổn. Sau cùng cũng có thể can thiệp ở vài thoát vị đĩa nhỏ để lấy đi những mảnh xương đè ép dây thần kinh : đó là một giải pháp thay thế rất đáng lưu ý đối với ngoại khoa khi phẫu thuật không thể thực hiện bởi vì sự phục hồi nhanh hơn và những biến chứng lâu dài ít hơn. Những thầy thuốc X quang đang nghiên cứu một kỹ thuật mới đẩy hứa hẹn bởi vì nó tránh đi xuyên qua da nhờ một ống thông : đó là phá hủy những u xương bằng một chùm siêu âm rất hội tụ, dưới sự kiểm soát của IRM.”
(LE FIGARO 8/12/2014)
2/ QUANG TUYẾN CAN THIỆP : ” TÔI ĐÃ HỒI PHỤC NHANH HƠN NHIỀU”
Trước khi được đề nghị một động tác quang tuyến điều trị, Hervé Leroux, 50 tuổi (Calvados), đã có thời gian trắc nghiệm những liệu pháp khác : đã gần 10 năm ông ta phải đương đầu với những vấn đề của các khối u ung thư của ống tiêu hóa.
” Tất cả đã bắt đầu năm 2005 bởi những cơn đau bụng không chịu nỗi Sau khi đã gọi một thầy thuốc cấp cứu, ông ta đã chẩn đoán cho tôi một tắc ruột và đã cho tôi nhập viện. Sự tắc nghẽn này được gây nên bởi một ung thư của ống tiêu hóa : do đó tôi đã chịu một cuộc mổ đầu tiên để lấy đi phần bị bệnh của ruột non và đại tràng sigma. Năm 2008, chính những tế bào ung thư ở bàng quang khiến tôi phải bị cắt bỏ một phần (cystectomie partielle : cắt bỏ một phần của bàng quang bằng phẫu thuật). Tuy nhiên thời gian hưu chiến ngắn ngủi bởi vì năm 2009, lần này chính những cục u ung thư (nodule cancéreux) ở gan mà người ta đã tìm thấy. Do đó một cuộc mổ được lên chương trình và người ta đã lấy một phần gan của tôi, điều này khiến tôi phải 2 tháng nghỉ việc và ban nằm yên nghỉ.
” Than ôi, vào năm 2012, sự theo dõi gan của tôi đã cho thấy rằng những cục u mới đã xuất hiện. Tôi đã quyết định đi khám một thầy thuốc chuyên về ung thư (oncologue) khác. Với ông ta, chúng tôi đã gợi ý giải pháp quang tuyến điều trị và trong lúc chờ đợi, ông ta đã đề nghị tôi thực hiện một scanner mỗi 3 tháng để xem các cục u lớn lên hay không. Vì hai cục u ở gan có khuynh hướng lớn lên, nên năm 2014, cuối cùng người ta quyết định thực hiện cho tôi một chimio-embolisation để tiêu hủy chúng.
” Tôi nhập viện chiều hôm trước khi mổ ở CHU de Caen và buổi sáng người ta đã điều trị tôi. Tôi chỉ hơi mệt, hơi buồn nôn, nhưng không đau và không sốt. Chỉ sau 5 ngày, tôi đã có thể trở về nhà. Tôi chỉ chịu một tình trạng mệt nhẹ dai dẳng trong 3 tuần. So với cuộc mổ trước đây của tôi có sự khác nhau : lần này, tôi không phải chịu những ống thông xuất ra từ khắp bụng của tôi, tôi không mang sẹo và một cách dễ dàng hơn, với một kết quả tốt vì lẽ hai cục u tạo vấn đề đã tan biến đi ! Đối với tôi, kỹ thuật này (do tính chất nhắm đích của nó) cũng có ưu điểm là tránh một số những tác dụng không được mong muốn và đáng sợ khi đó là hóa trị cổ điển.
” Từ những can thiệp này, tôi ghi nhận rằng phải, trong chừng mực có thể được, là người chủ chốt của ” đoạn đường chiến binh” chống lại căn bệnh của mình, không do dự nêu ra những câu hỏi và đòi hỏi một ý kiến thứ hai khi ta không nhận được những thông tin mong muốn, bởi vì thường có nhiều lựa chọn. ”
(LE FIGARO 8/12/2014)
3/ NHỮNG NHẬP VIỆN NGẮN NGÀY HƠN
Mặc dầu ý tưởng emboliser (bít một mạch máu) có thể làm sợ hãi một số người, đối với GS Jean-Pierre Pelage (CHU Caen), từng trải qua những bước đầu của embolisation những u xơ tử cung vào năm 1995 trong khi ông còn là nội trú ở bệnh viện Lariboisière (Paris), kỹ thuật này ngày nay rất được nắm vững.
” Trên thực hành, thầy thuốc X quang thực hiện một thăm dò chụp hình ảnh (siêu âm vùng chậu, nếu cần được bổ sung bởi một IRM) để xác định số lượng những u xơ, kích thước và vị trí của chúng. Điều đó cần thiết để đảm bảo tính khả thi của điều trị. Một khi được quyết định, can thiệp được lên chương trình (48 giờ nhập viện). Can thiệp được diễn ra với gây tê tại chỗ và nhằm bít bằng embolisation những động mạch tử cung nuôi dưỡng các u xơ. Sau embolisation, một thuốc giảm đau được tiêm truyền để tránh đau đạt cao điểm trong 6 đến 12 giờ sau can thiệp. Nghỉ việc một tuần (so với ít nhất 3 tuần sau phẫu thuật) và vài ngày điều trị giảm đau và kháng viêm là đủ. Cuối cùng, emboliser một u xơ như thế là một giải pháp thay thế myomectomie multiple (ngoại khoa bảo tồn cổ điển của u xơ) hay cắt bỏ tử cung (hystérectomie) bởi vì nó cho phép người đàn bà bảo tồn tử cung của mình”.
Nếu các bệnh nhân được điều trị bởi quang tuyến điều trị nhận thấy một cách nhanh chóng ở đó những lợi ích khi có thể thực hiện về mặt kỹ thuật (một nhập viện ngắn ngày hơn và một thời gian dưỡng bệnh nhanh hơn, cũng có lợi ích về mặt kinh tế. ” Thật vậy, những kỹ thuật vi xâm nhập này cho phép, đối với những chỉ định rất chính xác, những thời gian nhập viện ngắn ngày hơn so với ngoại khoa, với những kết quả tương đương. Đó chính là giải pháp duy nhất đối với những bệnh nhân không mổ được vì những bệnh khác làm họ có sức khỏe quá kém “, BS Audrey Fohlen (CHU de Caen) đã giải thích như vậy.
Phải cần nhiều thập niên để các nhà làm chính trị hiểu được lợi ích kinh tế của chirurgie ambulatoire. Vẫn còn hy vọng rằng không phải chờ đợi lâu như thế để họ sau cùng quan tâm đến những khả năng của quang tuyến điều trị.
(LE FIGARO 8/12/2014)
4/ TẠI SAO ĐỀ NGHỊ GHÉP THẬN
Docteur Valérie Moal
Centre de néphrologie et de transplantation rénale
Hôpital de La Conception (Marseille)
Ghép thận (transplantation hay greffe rénale) khi có thể thực hiện, mang lại cho bệnh nhân bị suy thận mãn tính giai đoạn tận cùng (nói một cách khác một sự ngừng hoạt động của hai quả thận) một hy vọng sống (espérance de vie) tốt hơn và một chất lượng sống tốt hơn những phương pháp thẩm tách khác nhau. Vì thế, năm 2014, ghép thận là điều trị lựa chọn của những bệnh nhân này và phải được đề nghị, bệnh nhân toàn quyền chấp nhận hay từ chối.
Năm 2012, ở Pháp, số những bệnh nhân bị một suy thận mãn tính giai đoạn tận cùng trên 70.000 : khoảng 40.000 được điều trị bằng thẩm tách máu (hémodialyse), 3000 bằng thẩm tách phúc mạc (dialyse péritonéale) và hơn 30.000 bệnh nhân đuợc ghép thận. Số những trường hợp ghép thận tùy thuộc vào số những transplant có sẵn. Để thực hiện một ghép thận, phải lấy một quả thận ở một người hiến. Sự lấy thận được thực hiện ở một người hiến tim còn đập (donneur à coeur battant) trong trạng thái chết não hay ở một người hiến tim ngừng đập (donneur à coeur arreté), đó là ghép từ người hiến chết (donneur décédé). Chết não (Mort cérébrale) chiếm dưới 2% những trường hợp tử vong. Sự hiếm hoi của biến cố này giải thích tình trạng khan hiếm thận. Những trường hợp lấy thận của những người hiến tim ngừng đập lại còn hiếm hơn bởi vì được thực hiện trong vài cơ sở được cho phép. Trong khi số những trường hợp lấy thận từ những người hiến chết vẫn bị hạn chế, số những người chờ thận gia tăng mỗi năm và trên 10.000 năm 2014.
Sự lấy thận có thể được thực hiện ở một người tình nguyện theo những điều kiện của loi de bioéthique năm 2004, đó là ghép từ người hiến còn sống. Thời gian sống của thận ghép ở người nhận khi đó tốt hơn so với khi thận phát xuất từ một người hiến chết. Ưu điểm thứ hai là sự thực hiện ghép thường có thể thực hiện trước khi bắt đầu điều trị bằng thẩm tách (greffe préemptive) bởi vì lấy và ghép được lên chương trình. Ngoài ra greffe péremptive còn cải thiện những kết quả của điều trị này.
Ở Pháp, sự hiến của người sống là không tốn tiền, tự do thỏa thuận và trong lợi ích điều trị của một người nhận được xác định. Mẹ, cha, anh chị em, con, ông nội ông ngoại, chú, cô, anh em chú bác, vợ hay chồng của một bệnh nhân, tất cả đưa ra bằng cớ có một cuộc sống chung ít nhất hai năm với người nhận, tất cả có thể đưa rằng bằng cớ một liên hệ tình cảm chặt chẽ và ổn định từ ít nhất hai năm với người nhận có thể đề nghị hiến thận.
Người hiến tự nguyện phải có sức khỏe tốt và những thăm dò khác nhau của bilan tiền phẫu nhằm chứng tỏ điều đó. Sự chọn lọc những người hiến được thực hiện nghiêm túc để cho phép người nhận và người hiến có một chức năng thận tối ưu sau khi ghép và hiến. Những thăm dò cũng phải chứng tỏ rằng người hiến và người nhận không bất tương hợp (incompatible) để tránh sự thải bỏ (rejet). Nhưng, ngay cả trong tình huống này, dự án hiến thận có thể được theo đuổi. Thật vậy, chúng ta hãy lấy thí dụ hai cặp hiến-nhận : Bà A muốn hiến cho chồng mình một trái thận nhưng chúng không tương hợp, và ông B muốn hiến cho con trai mình một trái thận nhưng chúng cũng không tương hợp, vậy một sự hiến ở mỗi cặp là không thể thực hiện được. Nếu bà A tương hợp với con trai của ông B và nếu ông B tương hợp với ông A, khi đó những hiến chéo (don croisé) này có thể xét đến. Trong tất cả các trường hợp, cặp hiến-nhận nhận một thông tin hoàn chỉnh. Người hiến sau đó được nghe bởi một ủy ban “người hiến sống” (donneur vivant). Ủy ban sẽ kiểm tra là người hiến đã hiểu rõ thông tin về những nguy cơ và hậu quả của sự lấy thận và đảm bảo người hiến toàn quyền quyết định. Sự lấy thận chỉ xảy ra sau khi đã được cho phép bởi ủy ban “người hiến sống” và sau khi người hiến đã bày tỏ sự đồng ý hiến thận của mình trước chủ tịch của tòa án thượng thẩm.
Câu chuyện của một cuộc ghép thận từ người hiến sống vừa đơn giản vừa phức tạp, phát xuất từ lòng quảng đại và tự nhiên của một người hiến thương yêu một người nhận và pha lẫn những khía cạnh khoa học và luân lý mà kíp nội-ngoại khoa hết lòng tôn trọng, để câu chuyện này là đẹp nhất.
(LE FIGARO 17/11/2014)
5/ THẨM TÍCH MỘT NGƯỜI GIÀ NHƯ THẾ NÀO ?
Professeur Philippe Brunet
Professeur Yvon Berland
Centre de néphrologie et de transplantation rénale
Université Aix-Marseille
Hôpital de la Conception
Thẩm tách (dialyse) ở người rất già, điều đó có thể là câu chuyện của Jean, 85 tuổi. Từ vài tháng nay, Jean mệt mỏi thường trực và ăn mất ngon. Thầy thuốc gia đình thực hiện cho ông một bilan máu và đã nói với ông đó là một suy thận. Ông đi khám một néphrologue, nghĩa là một thầy thuốc chuyên về những bệnh thận.
15 ngày sau, Jean đến thầy thuốc chuyên về thận (néphrologue). Vị thầy thuốc này giải thích với Jean rằng ông bị suy thận mãn tính. Những trái thận của ông không còn lọc một cách bình thường nữa. Chúng thải nước nhưng thải độc chất kém hơn. Khả năng lọc thận được ước tính 10% bình thường, điều này muốn nói rằng đó là một suy thận mãn tính giai đoạn tận cùng. Các quả thận của Jean dần dần ngừng hoạt động từ hôm nay cho đến 6 đến 12 tháng.
3 tuần sau, Jean đã làm một bilan mới xác nhận bilan đầu tiên. Có nhiều khả năng điều trị. Khả năng đầu tiên là ghép thận. Đó là giải pháp tốt nhất. nhưng Jean đã 85 tuổi và ông cũng bị suy tim sẽ làm cho cuộc mổ ghép thận rất khó. Thầy thuốc chuyên khoa thận giải thích rằng những bệnh nhân 80 tuổi có thể dược ghép thận, nhưng các cơ quan khác của họ phải hoàn toàn lành mạnh. Giải pháp thứ hai là thẩm tách. Đó là một phương pháp cho phép lọc máu để loại bỏ những độc chất. Có hai kỹ thuật thẩm tách, thẩm tách phúc mạc (dialyse péritonéale) và thẩm tách máu (hémodialyse).
Đối với thẩm tách phúc mạc, đó là một thẩm tách qua bụng. Tuyau nhỏ (hay cathéter) được đặt trong bụng. Nó đi ra trên thành bụng. Cathéter này được che phủ bởi một pansement nhỏ và không thể thấy được dưới lớp quần áo. Cathéter được sử dụng nhiều lần mỗi ngày để tiêm 2 lít dịch vô trùng vào trong bụng. Dịch này sẽ tích chứa những độc chất và sau vài giờ, nó sẽ được lấy ra, vứt bỏ và thay thế bởi một dịch mới. Điều trị này có thể được thực hiện ở nhà. Thủ thuật nhẹ nhàng và chịu được rất tốt.
Còn thẩm tách máu (hémodialyse) là một kỹ thuật lấy một cách trực tiếp các độc chất từ máu. Máu đi qua một máy sẽ tẩy sạch nó. Jean phải thực hiện mỗi tuần 3 buổi thẩm tách máu 4 giờ mỗi buổi trong một centre de dialyse. Bất hạnh thay trung tâm này nằm cách nhà của Jean 30 km, điều này làm ông lo ngại. Khi đó thầy thuốc chuyên khoa thận nói với ông về thẩm tách máu tại nhà (hémodialyse à domicile), với những máy nhỏ rất đơn giản. Nguyên tắc dựa trên những buổi thẩm tách máu ngắn hạn, hai giờ rưỡi, mỗi ngày.
Các bệnh nhân học tự đặt lấy các ống hay cathéter. Tuy nhiên, Jean sợ bị mệt và sợ không có kiên nhẫn để thực hiện tất cả những động tác này. Ngoài ra, ông sống đơn độc, trong khi cần có một người đi kèm để thực hiện thẩm tách máu tại nhà.
Jean đã suy nghĩ kỹ. Những đề nghị điều trị cho ông quá gò bó. Thầy thuốc chuyên khoa thận khi đó đề nghị với ông một consultation mới trước sự hiện diện của các con ông, không đồng ý với quyết định của ông không chịu làm thẩm tách. Thầy thuốc chuyên khoa thận khi đó đề nghị với Jean một suivi “à la carte” dành sự thoải mái của ông làm ưu tiên. Jean sẽ được theo dõi đều đặn bởi thầy thuốc chuyên khoa thận và thầy thuốc điều trị. Những loại thuốc sẽ được thích ứng kỹ càng, ông sẽ nhận những lời khuyên về chế độ ăn uống. Nhưng ông chắc chắn rằng thời gian sống dài hơn đối với những bệnh nhân theo một chương trình thẩm tách chuẩn, đồng thời có một sự giảm quan trọng chất lượng sống.
Trường hợp của Jean phải chăng là ngoại lệ ? Không. Trong số 41.000 bệnh nhân được thẩm tách hiện nay, ở Pháp, 6000 người 85 tuổi hoặc hơn, hoặc 15%. Số những bệnh nhân được thẩm tách trên 85 tuổi đã gia tăng hơn 50% giữa năm 2008 và 2012. Hiện tượng này có lẽ sẽ được tiếp tục ; điều này được giải thích bởi sự gia tăng và lão hóa của dân số. Vậy mục tiêu hiện nay : một mặt làm cho những kỹ thuật thẩm tích uyển chuyển hơn và dễ chịu đựng hơn và mặt khác phát triển ghép thận đối với những người có sức khỏe kém hơn.
(LE FIGARO 17/11/2014)
6/ UNG THƯ ĐẠI TRÀNG : NHỮNG VŨ KHÍ MỚI ĐỂ PHÒNG NGỪA VÀ ĐIỀU TRỊ.
GS David Khayat, trưởng khoa ung thư học của bệnh viện la Pitié-Salpetrière, tác giả của ” Prévenir le cancer, ça dépend aussi de vous”, édition Odile Jacob.
Hỏi : Ông hãy nhắc lại cho chúng tôi tần số ung thư đại tràng ở Pháp.
GS David Khayat : Ta thống kê 43.000 trường hợp mỗi năm trong đó, bất hạnh thay 16.000 đến 17.000 trường hợp tử vong. Còn về tỷ lệ mắc bệnh, ung thư có thể tránh được này đứng thứ ba sau ung thư vú và ung thư tuyến tiền liệt.
Hỏi : Những nguyên nhân là gì ?
GS David Khayat : 5% có nguồn gốc di truyền, những nguyên nhân khác nói chung do một chế độ ăn uống xấu, do nghiện thuốc lá…Trong hầu hết các trường hợp, sự tạo thành một polype xảy ra trước khối u, do đó tầm quan trọng của điều trị phát hiện ở một giai đoạn sớm để lấy đi polype càng sớm càng tốt !
Hỏi : Khi ta nói chế độ ăn uống xấu, điều đó hơi mơ hồ…Có những yếu tố làm dễ hay không ?
GS David Khayat : Không có chế độ ăn uống phổ quát. Nhưng sự tiêu thụ quá mức charcuterie, nhất là công nghiệp, sự cung cấp không đủ các sợi, các chất chống oxy hóa là có hại.
Hỏi : Làm sao phát hiện những triệu chứng đầu tiên của một ung thư đại tràng ?
GS David Khayat : Chúng khá bình thường và có thể giống với một vấn đề ruột khác, như những rối loạn transit, sự hiện diện của máu trong phân, một sự rất mỏi mệt. Thường nhất đó là những hemorroides hay những rối loạn tiêu hóa. Trước sự dai dẳng của các triệu chứng, chính thầy thuốc kê đơn hay không một soi đại tràng, thăm dò chủ yếu phát hiện sự hiện diện của một polype hay ung thư. Nhưng ngay khi xuất hiện những dấu hiệu đầu tiên, tuyệt đối phải nhờ đến một xét nghiệm điều tra phát hiện.
Hỏi : Những ưu điểm của trắc nghiệm dịch tễ học mới đối với Hemoccult ?
GS David Khayat : Trắc nghiệm chính xác hơn, nhạy cảm hơn và dễ sử dụng hơn. Nó phát hiện đến hai lần rưởi nhiều hơn các ung thư và 4 lần các polype nhiều hơn so với hemoccult. Phương thức nhằm đặt rất ít phân trên một bandelette và gởi nó cho thầy thuốc. Trắc nghiệm mới này chỉ cần một mẫu nghiệm duy nhất, thay vì ba với Hemoccult. Ở phòng xét nghiệm, mẫu nghiệm được phân tích nhờ một lecteur automatisé rất hiệu năng, đảm bảo một tính trung thực rất lớn.
Hỏi : Những điều trị quy ước tùy theo những giai đoạn nghiêm trọng ?
GS David Khayat : Khi đó chỉ là một polype đã thoái hóa, điều trị chỉ là phẫu thuật. Nếu ung thư đã phát triển nhưng vẫn còn khu trú, ta bổ sung bằng một hóa trị. Ở một giai đoạn nào đó, ung thư đã có thể xâm nhập thành ruột và, có thể những hạch lân cận. Lại nữa, sau phẫu thuật, điều trị là một hóa trị. Nếu trực tràng bị xâm nhập, ta bổ sung bằng một xạ trị. Nếu ung thư đã di căn, xâm nhập những cơ quan khác, ngày nay chúng tôi có những điều trị bổ sung khác : những điều trị nhắm đích (thérapie ciblée).
Hỏi : Những tiến bộ mới nhất của những liệu pháp nhắm đích đối với những ung thư di căn.
GS David Khayat : Những thuốc mà chúng tôi sử dụng đã có nhiều tiến bộ ! Ngoài những thuốc cổ điển của hóa trị, đã xuất hiện những thuốc phong bế những huyết quản nuôi dưỡng các khối u như bévacimuzab (Avastin) và, mới đây, đối với những bệnh nhân đề kháng với điều trị này, aflibercept. Trong trường hợp thất bại của những hóa trị cổ điển, ta nhờ đến régorafénib (không còn bằng tiêm truyền nữa nhưng bằng đường miệng). Với loại thuốc sau hết này, một công trình nghiên cứu rộng rãi đã được thực hiện trên 760 bệnh nhân đã đề kháng với những hóa trị khác. Bệnh nhân vẫn được kiểm soát rất lâu với một chất lượng sống tốt.
Hỏi : Đối với những ung thư di căn này, những tiến bộ mới nhất là gì ?
GS David Khayat : Đó là sự hiệu chính của panitumumab (Vectibix), một loại thuốc tăng cường tác dụng của hóa trị ở vài bệnh nhân (được chọn lọc bằng một trắc nghiệm lên khối u của họ), như một kháng thể mới. Trong 30% các trường hợp, một thử nghiệm lớn trên hơn 400 bệnh nhân đề kháng với tất cả các hóa trị đã chứng minh một sự ổn định của căn bệnh. Ngày nay, tính hiệu quả của những điều trị khác nhau đến độ nó cho phép một phẫu thuật các ung thư di căn ở khoảng 20% các bệnh nhân không mổ được, đặt họ trong tình trạng thuyên giảm hoàn toàn.
(PARIS MATCH 26/3/2015-1/4/2015)
7/ CÁC THẦY THUỐC BỊ TRẦM CẢM (BLUES)
Thảm kịch xảy ra ở khoa sản Orthez đưa ra ánh sáng một hiện tượng bị quên lãng : các nhân viên điều trị, bị nghiện ngập, bị burn-out và tự tử, đã không được điều trị tốt.
PRATICIENS. ” Tôi sẽ săn sóc điều trị (…) cho bất cứ ai sẽ yêu cầu tôi điều đó (…). Bệnh nhân và các đồng nghiệp của tôi dành sự quý mến nơi tôi nếu tôi trung thành với những lời hứa của tôi ; tôi sẽ bị ô danh và khinh bỉ nếu tôi không tròn lời hứa.” Đó là lời thề Hippocrate, được đọc lên bởi những thầy thuốc trẻ.
Một lời thề không có giá trị pháp lý, nhưng khiến những thầy thuốc chịu một mệnh lệnh : nghệ thuật (art) của họ là một thiên chức (sacerdoce) không chấp nhận một khiếm khuyết (défaillance) nào. Thảm kịch Orthez, vào cuối tháng 9 năm 2014 đã chứng kiến trường hợp tử vong của một phụ nữ trẻ khi được mổ césarienne và sự hỏi cung một thầy thuốc gây mê, dọi lên một ánh sáng mới về một thực tế quá thường bị che dấu : hơn những ai khác, những người điều trị đau khổ do công việc của họ. Và hơn những ai khác họ đau khổ một cách thầm lặng.
Số các thầy thuốc tự tử 2,37 lần nhiều hơn dân thường, theo một công trình nghiên cứu được thực hiện năm 2003 bởi Caisse autonome de retraite des médecins de France (CARMF), thầy thuốc tự tử “điển hình” là một phụ nữ 48 tuổi, thầy thuốc tâm thần, đa khoa hay gây mê.
” Tỷ lệ lưu hành của chứng trầm cảm ở các thầy thuốc gần giống với dân thường, nhưng các thầy thuốc “thành công” tự tử tốt hơn bởi vì biết các loại thuốc”, BS Max-André Doppia, thầy thuốc của CHU de Caen và chủ tịch của commission Smart (Santé du médecin anesthésiste-réanimateur au travail) đã nói như vậy.
Các thầy thuốc cũng không thoát khỏi chứng nghiện ngập. Vào năm 2005 Max-André Doppia đã tiến hành một công trình nghiên cứu ở các thầy thuốc gây mê-hồi sức : 10,9% những thầy thuốc được hỏi đã lạm dụng và/hoặc phụ thuộc ít nhất một chất hướng thần (psychoactive) khác với thuốc lá, đứng đầu là rượu (đối với 59% trong số họ), tiếp theo là những thuốc an thần và thuốc ngủ (41%). ” Tuy nhiên không nên nghĩ rằng các thầy thuốc gây mê bị liên hệ hơn là những thầy thuốc khác. Một cách đơn giản, chúng ta là những người duy nhất đã dám đề cập vấn đề. Có lẽ bởi vì chúng ta có một văn hóa về nguy cơ rất sâu”.
Còn về nguy cơ burn-out, cũng được gọi là syndrome d’épuisement professionnel, theo nhiều công trình nghiên cứu, nó liên hệ 40 đến 60% những thầy thuốc của tất cả các chuyên khoa. Một công trình, được trình bày ở hội nghị mới đây của Société européenne d’oncologie médicale, cho thấy rằng 71% những thầy thuốc chuyên về ung thư trẻ có ít nhất một yếu tố épuisement professionnel. ” Các thầy thuốc chuyên ung thư, BS Susana Barjee, người lãnh đạo công trình nghiên cứu, có những quyết định phức tạp về sự quản lý ung thư, theo dõi sự sử dụng những thérapie toxique, làm việc nhiều giờ và liên tục đối diện với những bệnh nhân đau khổ và chết.”
Trò chơi buồn bã này với chết chóc và đau đớn thêm vào surcharge de travail, sự lo sợ bị sai lầm, sự quấy nhiễu của người bệnh, gánh nặng của hành chánh, sự không công nhận vai trò của thầy thuốc và ít chỗ dành cho đời sống cá nhân. Một stress, mặc dầu không phải là đặc quyền của các thầy thuốc. Nhưng họ nắm sinh mạng trong tay. ” Khi anh làm việc 54 giờ mỗi tuần, khi anh chạy khắp nơi bởi vì thiếu nhân viên, khi anh đơn độc và đương đầu với một tình huống lâm sàng phức tạp…Anh có nguy cơ bị một tai biến !”, Max-André Doppia đã tóm tắt như vậy.
Theo ông, kíp là một chỗ dựa (garde-fou) thiết yếu. ” Gây mê đã trở thành một trong những động tác an toàn nhất, nhưng với những planning mổ rất chargé, ta có thể cố đi nhanh hơn, làm nhiều hơn ; chính ở đó mà tập thể là quan trọng, để nhắc nhở chúng ta khi ta làm không tốt công việc.” Theo các chuyên gia, có 4 lần burn out nhiều hơn trong kíp mà sự gắn bó với nhau xấu. Trong thảm kịch Orthez, BS Doppia đặt câu hỏi nhất là về sự kiện chứng nghiện rượu của cô ta đã không được phát hiện bởi các đồng nghiệp.” Cô ta đã ở đó từ ba tuần, thế mà người ta không nói với chúng tôi gì hết về thời gian đó đã diễn ra như thế nào.” Điều đó không miễn trừ cô ta những trách nhiệm của mình, nhưng cơ sở tiếp nhận cô ta cũng phải có phần trách nhiệm, theo BS Doppia. ” Thật quá đơn giản khi đổ hết mọi chuyện cho sự yếu ớt của cá nhân.”
Ý thức rằng chất lượng điều trị một phần tùy thuộc vào chất lượng sống của các thầy thuốc, Bộ y tế xét đến chất lượng sống của các thầy thuốc lúc chứng nhận các cơ sở y tế.
Vào năm 2001, điều tra Sesmat cho thấy rằng chỉ 15% những thầy thuốc ứng viên burn-out ý thức điều đó. Một cách lạ kỳ, các thầy thuốc tự điều trị cho mình kém. ” Họ ưa thích tự chẩn đoán và tự cho thuốc hơn. Nhưng khi anh bắt đầu débloquer, anh không thể tiếp tục như vậy “, Max-André đã cảnh cáo như vậy.
Con về y khoa lao động, nó chỉ bắt buộc đối với những người ăn lương của bệnh viện và chỉ được lui tới bởi 40% trong số những người này. Đối với các thầy thuốc thực hiện trực gác và có khả năng bị mệt và stress, đó không phải là fantastique, BS Doppiua đã ghi nhận như vậy. Một thầy thuốc hành nghề tự do có thể đã treo bảng khám bệnh cách nay 40 năm và đã không bao giờ đo huyết áp của mình !” Cũng vậy, kíp đối đầu với một tai biến thường tự để buông trôi. Khi một cảnh sát viên dùng vũ khí công vụ của mình hay khi một nhân viên hỏa xa bị một tai nạn, cả một thủ tục được thiết đặt. Những người này được khám bởi một psychologue và không trở lại nhiệm sở tức thời, Max-André Doppia đã chứng thực như vậy. Đối với các thầy thuốc không gì được dự kiến cả. “The show must go on”
(LE FIGARO 13/10/2014)
8/ KHI NHỮNG NGƯỜI CHẾT THỨC DẬY
Sự “hồi sinh” (résurrection) của những bệnh nhân được tuyên bố là đã chết là do sai lầm chẩn đoán hơn là do phép nhiệm màu
REANIMATION. Ta nghĩ đến Roméo, vô cùng đau đớn trước thân xác bất động của Juliette. Mặc dầu được tuyên bố là đã chết, nhưng vài bệnh nhân tỉnh dậy trong khi chẳng ai còn ngờ nữa…Một hiện tượng ngoại lệ, nhưng không có gì là nhiệm màu.
Cách nay vài tuần, tim của một sản phụ người Mỹ 40 tuổi ngừng đập sau khi sinh. Bà ta bị một embolie amniotique : dịch ối (liquide amniotique) mà thai nhi nằm trong đó đã đi qua máu mẹ, các tế bào thai nhi di chuyển về những động mạch phổi và làm tắc nghẽn chúng. Sau 45 phút cố gắng hồi sinh vô ích, các thầy thuốc chuẩn bị tuyên bố khai tử. Nhưng, trái với mọi chờ đợi, tim của người mẹ trẻ đập lại một cách ngẫu nhiên. Vài ngày sau, báo chí Ba lan kinh ngạc thán phục về bà già 91 tuổi đã tỉnh dậy…ở nhà xác ! Hai câu chuyện này đặt ra một câu hỏi : làm sao người ta có thể “tỉnh dậy” trong khi đã bi khai tử trên phương diện lâm sàng ?
” Vài trường hợp “nhiệm màu” được kể trong những năm gần đây được giải thích bởi những sai lầm chẩn đoán liên quan đến những khám lâm sàng được thực hiện không điện tâm đồ”, GS Pierre-François Laterre, chủ tịch Société de réanimation de langue française. ” Trong trường hợp bà người Ba lan, ta có thể nghĩ hoặc là một ngộ độc thuốc, khiến bệnh nhân đi vào hôn mê sâu, hoặc một tình trạng hạ thân nhiệt (hypothermie). Khi đó người bệnh trong tình trạng đông miên (en hibernation), gần như không thở, mạch không còn có thể phát hiện nữa, thân thể giảm quá trình chuyển hóa.”
TÍNH CHẤT NHANH CHÓNG VÀ CHẤT LƯỢNG CỦA XOA BÓP TIM
Đó là điều có thể xảy đến ở những người bị té vào một vùng nước rất lạnh hay bị chôn vùi dưới tuyết ; lạnh sẽ bảo vệ các tế bào. Vào năm 2000, tạp chí y khoa The Lancet báo cáo trường hợp Anna Bagenholm, một thầy thuốc Thụy Điển đã làm hồi sinh không để lại di chứng, sau khi nạn nhân nằm dưới băng giá 45 phút sau khi bị té trong một cuộc đi dạo. Trường hợp này khá ngoại lệ nên được vinh dự công bố trên tạp chí khoa học uy tín. Bất hạnh thay, trong phần lớn các trường hợp nạn nhân đều bị chết : khi cơ thể được sưởi ấm trở lại, máu tưới các mô thiếu oxy. Các mô sẽ tiếp tục bị thiếu oxy và điều đó sẽ gây nên những tổn hại, đó là điều mà ta gọi là ischémie de reperfusion. Kết quả : những người được làm hồi sinh này sẽ chết một hay hai ngày sau vì những suy cơ quan (défaillance cardiaque).
Trường hợp của bà mẹ gia đình trẻ người Mỹ có hơi khác. Đó là một ngừng tim ở bệnh viện, được xử trí tức thời và trong những điều kiện tốt nhất. ” Có khi, nhưng là trường hợp ngoại lệ, tim đập trở lại trong khi chúng ta đã xuôi tay. Điều đó đã xảy đến một lần. Tôi nói rằng đó là một sai lầm của tuổi trẻ “, GS Benoit Misset, thầy thuốc chuyên khoa hồi sức, giảng viên ở Paris-V đã xác nhận như vậy. Trong số những chuyên gia được hỏi, ông ta là người duy nhất đã trải qua kinh nghiệm này.
Nhưng một điều chắc chắn, sự nhanh chóng mà một xoa bóp tim được thực hiện và chất lượng của xoa bóp làm gia tăng những khả năng sống còn. ” Với một xoa bóp tim, anh thay đổi dòng thời gian. Anh làm máu lưu thông, anh kéo dài thời gian sống còn của các tế bào và nhất là những tế bào của não. Mỗi phút trôi qua không xoa bóp tim, đó là 10% cơ may sống sót bớt đi “, GS Xavier Jouven, trưởng kíp ” Epidémiologie cardio-vasculaire et mort subite” trong unité Inserm 970, đã nói như vậy.
Vấn đề là đa số những trường hợp ngừng tim xảy ra ngoài bệnh viện, trong đường xá hay tại nhà. ” Ở Pháp, thường nhất những nhân chứng gọi cấp cứu nhưng không làm gì hết, trong khi ở các nước Bắc Âu, một xoa bóp tim được thực hiện một cách hệ thống trong 3 phút. Thế mà nếu không xoa bóp tim trong vòng dưới 5 phút những tổn hại ở não bộ là đáng kể “, BS Bruno Levy, thầy thuốc chuyên khoa hồi sức của Nancy đã xác nhận như vậy. Kết quả : chỉ 5% trong số 40.000 người Pháp bị quật ngã trên đường là sống sót không di chứng. Ở Seattle (Hoa Kỳ), tỷ lệ này đạt 20%. Nếu tỷ lệ sống sót ở Paris và vùng ngoại ô đã gia tăng nhẹ để đạt 7%, đó là nhờ một thực hiện xoa bóp tim thường hơn đối với những người hiện diện gần nạn nhân.
” HIẾN CƠ QUAN”
Song song, sự xử trí bởi các thầy thuốc cấp cứu càng ngày càng tích cực hơn, nhất là ở những người trẻ tuổi nhất. ” Hôm nay, chúng tôi xử trí tích cực hơn nhiều cho những người trẻ tuổi. Chúng tôi tiếp tục xoa bóp tim trên 40 phút. Nhưng tỷ lệ sống còn không vì thế mà tiến triển. Và nếu chúng tôi tiếp tục quá lâu một hồi sinh sẽ không cứu được nạn nhân, chúng tôi mất khả năng có được một hiến cơ quan”, GS Xavier Jouven đã chứng thực như vậy. Các chuyên gia đang xác định những tiêu chuẩn cho phép các kíp Samu xác định một cách rất nhanh chóng ai ” có thể cứu được” và ai không.
(LE FIGARO 31/12/2014)
9/ XỬ TRÍ NGỪNG TIM : CHÚNG TA ĐÃ ĐẾN ĐÂU RỒI ?
Professeur Pierre-Yves Gueugniaud
Anesthésiste-réanimateur
Directeur du Samu de Lyon
Pôle urgences-Réanimation médicale-Anesthésie-
Réanimation-Samu
CHU de Lyon
Ngừng tim (AR : arrêt cardiaque) là cấp cứu y khoa cực kỳ nhất mà ta có thể tưởng tượng. Để xử trí nó, hồi sức tim phổi (RCP : réanimation cardio-pulmonaire) “hiện đại” đã được mô tả vào đầu những năm 1960. Ngừng tim bất ngờ hay chết đột ngột của người lớn được xem là một vấn đề quan trọng của y tế công cộng trong những nước công nghiệp. Ở Pháp nó liên quan hơn 50.000 trường hợp ngừng tim mỗi năm. Nguyên nhân y khoa của ngừng tim là thường gặp nhất (khoảng 90% những ngừng tim), và hơn một nửa những ngừng tim y khoa này có một nguyên nhân tim.
Để nhạy cảm hóa các trí óc về vấn đề liên quan đồng thời những người hành nghề y tế và công chúng, các chuyên gia quốc tế từ hai mươi năm nay nhất trí quan niệm căn bản “chuỗi sinh tồn ” (chaine de survie). Đó là một chuỗi gồm 4 mắc xích biểu hiện 4 giai đoạn khác nhau của xử trí một ngừng tim : mắc xích đầu tiên là báo động (alerte) bởi nhân chứng đầu tiên, mắc xích thứ hai thể hiện những động tác cơ bản của sự sống còn (xoa bóp tim và sự bổ sung thông khí), mắc xích thứ ba là khử rung và mắc xích cuối cùng là réanimation médicalisée của ngừng tim và những kế tiếp của hồi sức tim phổi (RCP). Sự vắng mặt của một trong những mắc xích này làm tiêu tan mọi hy vọng thành công, và sự nhanh chóng thực hiện của mỗi trong những mắc xích được thiết đặt là một yếu tố chủ yếu của thành công của hồi sức tim phổi.
NHỮNG MÁY KHỬ RUNG, YẾU TỐ CHỦ YẾU THÀNH CÔNG HỒI SINH
Về phía đại chúng, báo động cho Centre 15 (Centre de régulation du Samu) là ưu tiên để cho phép khởi động không trì hoãn secours. Sự nhận biết ngừng tim bởi một nhân chứng không được đào tạo, được thực hiện, một mặt, bởi sự mất tri giác và không phản ứng, và, mặt khác, bởi sự ngừng thở hay sự hiện diện của một hô hấp “hấp hối” (respiration agonique). Sự lấy mạch không còn được khuyến nghị nữa. Khi đó phải bắt đầu tức thời hồi sức tim phổi bởi những ép ngực, được thực hiện liên tục, không gián đoạn, bằng cách đè lên phía giữa của ngực với các lòng bàn tay đặt lên nhau và những cánh tay duỗi thẳng đứng, bằng cách đè xương ức xuống ít nhất 5 cm với một tần số 100 mỗi phút. Nếu không được đào tạo những động tác cơ bản sống còn, những ép ngực này có thể được thực hiện không cần luân phiên bởi một thông khí nhân tạo bằng hà hơi thối ngạt miệng-miệng (bouche-à-bouche) cho đến khi những secours professionnel đến. Đó là message sẽ chuyển đến cho anh bởi régulation médicale.
Ngoài tầm quan trọng của những ép ngực, sự sử dụng những máy khử rung tự động ngoài (DAE : défibrillateur automatisé externe) bởi đại chúng là yếu tố cơ bản thứ hai đối với sống còn của những ngừng tim, nhất là khi nguyên nhân là tim.
Thật vậy, rung thất thường gặp nhất trong những trường hợp này có thể tiếp cận với sốc điện, cho phép phục hồi nhanh một hoạt động tuần hoàn tự nhiên (RACS : récupération d’une activité circulatoire spontanée) trong một số lớn các trường hợp. Vào lúc này, đó là yếu tố thành công chủ yếu của hồi sức tim phổi (RCP). Theo những dữ kiện Pháp phát xuất từ Registre électronique des arrêts cardiaques (RéAC), những cơ may sống còn vào lúc một tháng của một ngừng tim chuyển từ 5% khi không có DAE lên 15% khi có một DAE lúc RCP bởi những nhân chứng, và hơn 30% khi cùng DAE phát ra một sốc điện trước khi các pompier và Samu đến.
Dầu thế nào đi nữa, ở Pháp sự thông tin đại chúng vẫn không đủ. Điều mong muốn là cần phổ biến một cách rộng rãi hơn khẩu hiệu được đề nghị bởi Fédération française de cardiologie và Samu-Urgences de France : ” Arrêt cardiaque : 3 động tác cho một cuộc đời = Gọi-Xoa bóp-Khử rung.”
Ở Pháp, secours đầu tiên được thực hiện trong đại đa số các trường hợp bởi các lính cứu hỏa (sapeur-pompier). Nó sẽ cho phép kéo dài hồi sức tim phổi cơ học (RCP mécanique) bằng cách thiết lập một thông khí nhân tạo nhờ một insufflateur manuel, cho phép một thông khí có chất lượng với oxy. Chuỗi hồi sức tim phổi cơ học khi đó là 30 compression đối với 2 insufflation. Ngoài ra, những người chuyên nghiệp về sơ cứu này có thể thực hiện những sốc điện nếu cần, tất các các kíp lính cứu hỏa ở Pháp đều có DAE.
Vai trò của thầy thuốc cấp cứu trong thiết đặt một RCP spécialisée gồm ba phần. Trước hết cải thiện chất lượng và tính hiệu quả của RCP mécanique đã được thực hiện bởi những secouriste. Như thế oxygénation mô sẽ có thể được cải thiện bởi nội thông khí quản bệnh nhân. Sau đó khi RACS (sự phục hồi một hoạt động tuần hoàn tự nhiên) không tùy thuộc vào choc điện, khi đó sự sử dụng adrénaline là phương tiện duy nhất cho phép RACS này. Mặc dầu điều trị này vẫn có tính hiệu quả hạn chế, adrénaline là thuốc duy nhất được khuyến nghị.
Sau cùng, một khi đã có được RACS, thầy thuốc của Samu vẫn còn cần thiết để đảm bảo sớm sự ổn định các chức năng sinh tồn và sự kiểm soát những suy cơ quan, nhất là tim mạch và não. Như thế, sự chống lại tăng thân nhiệt, thậm chí sự thiết đặt một giảm thân nhiệt vừa phải là điều mong muốn.
Vào năm 2015, sự cải thiện điều trị của những ngừng tim trước hết được chứng tỏ bởi sự phân tích của những thực hành của chúng ta qua những registre national. Registre français (“ReAC”) hôm nay có gần 40.000 hồ sơ cho phép nêu lên tính hiệu quả trong xử trí những ngừng tim của hệ thống Pháp kết hợp sơ cứu nhanh bởi những sapeurs-pompiers và những thuốc cấp cứu bởi Smur.
(LE FIGARO 2/3/2015)
10/ TƯƠNG TÁC THUỐC : NHỮNG NGUY CƠ.
BS Patrick Errard, chủ tịch các Xí nghiệp dược phẩm, cảnh giác những nguy cơ do những sai lầm khi sử dụng, đặc biệt ở những người già.
Hỏi : Ông vừa tái xuất bản tác phẩm ” 100 câu hỏi về thuốc”. Bao nhiêu người bị hay là nạn nhân mỗi năm những tai biến liên quan đến sự sử dụng một loại thuốc.
BS Patrick Errard. Ở Pháp ta liệt kê khoảng 8000 đến 12.000 người, trong đó 5 đến 10% phải được nhập viện.
Hỏi : Trong số những phối hợp có hại, ông hãy kể cho chúng tôi những phối hợp thông thường nhất.
BS Patrick Errard. Thí dụ, những người ta kê đơn cho họ 3 loại thuốc phai dùng trong ngày, một để điều trị chứng lo âu, thuốc thứ hai để điều trị chứng trầm cảm và thuốc thứ ba để điều trị dị ứng, đôi khi gây sai lầm khi dùng chúng cùng lúc vào lúc đi ngủ, gây nên một nguy cơ bị malaise và té ngã khi họ đứng dậy vào ban đêm, những tai biến rất thường dẫn họ đến cấp cứu. Phải biết rằng một hỗn hợp không thích hợp của nhiều loại thuốc có thể làm gia tăng mạnh những tác dụng của một loại thuốc khác. Một trường hợp khá thường gặp là trường hợp những bệnh nhân đang được điều trị với thuốc kháng đông (anticoagulant) nhờ đến aspirine để làm giảm đau đầu. Phối hợp này có thể tạo điều kiện cho sự xuất hiện của những trường hợp xuất huyết. Thí dụ khác : trường hợp những người theo một điều trị hạ huyết áp dùng đồng thời một thuốc khác có cùng tác dụng. Những người này rất có nguy cơ bị trụt huyết áp đột ngột. Tôi trấn an các bệnh nhân : mỗi loại thuốc trên thị trường có những đảm bảo an toàn và hiệu quả được đòi hỏi.
Hỏi : Có phải tất cả đều phản ứng cùng cách với những hỗn hợp bị chống chỉ định này ?
BS Patrick Errard : Mặc dầu những hiểm nguy hiện hữu, chúng được biểu hiện một cách rất thay đổi tùy theo tính nhạy cảm của mỗi người.
Hỏi : Những trường hợp nào làm dễ những tai biến ?
BS Patrick Errard : 1. Tuổi tác : lúc ta già di, có thể thuốc thải ít tốt hơn. Thời gian sống của thuốc trong máu không luôn luôn giống nhau. 2. Số lượng : càng có nhiều thuốc được kê đơn trên một toa thuốc, càng có nhiều nguy cơ sai lầm trong sử dụng. 3. Những rối loạn trí nhớ : chúng có thể gây nhưng quên lãng.
Hỏi : Hãy chỉ cho chúng tôi những rối loạn phải khiến đặc biệt báo động ?
BS Patrick Errard : Sự xuất hiện chóng mặt khi đứng dậy hay khi thay đổi tư thế, một cảm giác không vững, lú lẫn, sự xuất hiện xuất huyết dưới da, dưới lợi răng.
Hỏi : Có phải người ta đã chứng thực rằng những tai biến gây nên bởi những phối hợp thuốc là thường gặp hơn với những générique ?
BS Patrick Errard : Những tương tác giữa các loại thuốc, dầu biệt dược (produit de marque) hay générique, đều giống nhau. Nhưng những người già có thể nhạy cảm hơn đối với sự thay đổi tên thuốc. Chúng ta hãy nhắc lại vụ, rất được phổ biến bởi giới truyền thông, về một bà già đã lầm lẩn một thuốc ngủ với một thuốc lợi tiểu…
Hỏi : Dưới dạng nào, những sai lầm liều lượng thường gặp nhất ?
BS Patrick Errard : Phải cảnh giác với những thuốc uống (thuốc giọt, sirops…) hơn là với những thuốc viên. Đó là lý do tại sao công nghiệp dược đã hiệu chính những système “unidose”.
Hỏi : Theo ý kiến của ông, việc bán thuốc theo đơn vị (vente de médicaments à l’unité) có làm giảm những nguy cơ này không ?
BS Patrick Errard : Còn quá sớm để có thể phát biểu. Mục đích trước hết là tạo điều kiện cho một sự tuân thủ các điều trị tốt hơn. Thí nghiệm đang được tiến hành.
Hỏi : Những lời khuyên của ông cho những người dùng thuốc mỗi ngày là gì ?
BS Patrick Errard : 1. Thông tin tốt thầy thuốc kê đơn chuẩn bị đơn thuốc mà người ta dùng. 2. Đừng do dự hỏi thầy thuốc của mình, thậm chí dược sĩ, có những nguy cơ tương tác giữa các thuốc được kê đơn không. 3. Trong trường hợp thay thế bởi một générique, yêu cầu dược sĩ định rõ cho mình nó tương ứng với loại thuốc nào được ghi trên đơn thuốc. 4. Hãy tôn trọng những giờ giấc dùng thuốc để không bị một sự tích tụ thuốc trong máu và do đó một nguy cơ tác dụng quá quan trọng. Hãy biết rằng tự gia tăng những liều lượng có thể gây nên một ngộ độc. 5. Chuẩn bị trong một hộp thuốc những thuốc phải dùng trong ngày hay tuần. Sử dụng đối với những người già những mốc cố định (bữa ăn, buổi truyền hình…).
(PARIS 29/1/2015-4/2/2015)
BS NGUYỄN VĂN THỊNH
(14/4/2015)

Cấp cứu ngoại thần kinh số 8 – BS Nguyễn Văn Thịnh

TEST 1
XUẤT HUYẾT MÀNG NÃO
(HÉMORRAGIE MÉNINGÉE)
Adam J.Rosh, MD
Assistant Professor
Department of Emergency Medicine
Wayne State University School of Medicine
Detroit, MI
Một người đàn ông 57 tuổi với một tiền sử cao huyết áp và migraine đến phòng cấp cứu kêu đau đầu, bắt đầu cách nay 2 ngày. Ông ta nói rằng đau đầu bắt đầu đột ngột với cường độ cực đại trong khi anh ta đang đại tiện. Đau xảy ra liên tục đặc biệt ở vùng chẩm và được liên kết với cứng cổ nhẹ và sợ ánh sáng nhẹ. Anh phủ nhận không bị sốt gần đây. Một chụp cắt lớp vi tính đầu không tiêm chất cản quang được thực hiện và bình thường. Biện pháp nào sau đây là thích hợp nhất trong điều trị ?
a. Cho metoclopramine để giảm mửa và ketorolac để điều trị đau
b. Chọc dò tủy sống
c. Điều trị thực nghiệm (empiric treatment) đối với viêm màng não bằng kháng sinh tĩnh mạch
d. Mannitol tĩnh mạch để hạ áp lực nội sọ
e. Chụp động mạch não để tìm một phình mạch
Đáp :
Câu trả lời đúng là b. Bệnh nhân có một bệnh sử lâm sàng phù hợp với một xuất huyết dưới nhện. Chụp cắt lớp vi tính não không tiêm chất cản quang là thủ thuật lựa chọn để chẩn đoán xuất huyết dưới nhện và nên được thực hiện ở bất cứ bệnh nhân nào với một khởi đầu mới của một triệu chứng đau đầu dữ dội hay dai dẳng. Chụp cắt lớp vi tính có một độ nhạy cảm 95% để phát hiện xuất huyết dưới nhện. Nếu chụp cắt lớp vi tính âm tính, một chọc dò tủy sống nên được thực hiện bởi vì vài bệnh nhân với xuất huyết dưới nhện có một chụp cắt lớp vi tính bình thường. Một dịch vàng supernatan (xanthochromia), có được bằng quay ly tâm một mẫu nghiệm dịch não tủy có máu, có thể giúp phân biệt xuất huyết dưới nhện với xuất huyết do chọc dò gây chấn thương (traumatic tap). Nếu chẩn đoán vẫn còn là một vấn đề, một chụp mạch não có thể được đòi hỏi.
(a) Cho metoclopramide và ketorolac là hữu ích trong điều trị đau do hội chứng migraine. Do hoạt tính kháng tiểu cầu, ketorolac (Taradyl,Toradol) và những thuốc kháng viêm không phải sterod khác (AINS) bị chống chỉ định ở những bệnh nhân có thể đang chảy máu tích cực.
(c) Điều trị viêm màng não với các kháng sinh bằng đường tĩnh mạch không nên trì hoãn nếu chẩn đoán được nghi ngờ. Tuy nhiên bệnh sử của bệnh nhân không phù hợp với chẩn đoán này (bệnh nhân không sốt và không có những triệu chứng toàn thân) và chọc dò tủy sống sẵn sàng.
(d) Tiêm truyền mannitol tĩnh mạch làm hạ khẩn cấp tăng áp lực nội sọ hay khi nghi ngờ tụt kẹt sắp xảy ra (impending herniation).
(e) Chụp động mạch não là tiêu chuẩn vàng để chẩn đoán phình mạch não nhưng chọc dò tủy sống phải được thực hiện để xác nhận sự hiện diện của xuất huyết nội sọ trước khi chụp cắt lớp vi tính với chất cản quang.
Emergency Medicine : PreTest
BS NGUYỄN VĂN THỊNH

(13/4/2015)Cấp cứu Nội Tiết + Chuyển Hoá số 24 – BS Nguyễn Văn Thịnh

TEST 1
HẠ ĐƯỜNG HUYẾT
(HYPOGLYCEMIA)
Adam J.Rosh, MD
Assistant Professor
Department of Emergency Medicine
Wayne State University School of Medicine
Detroit, MI
Một người đàn bà 69 tuổi với tiền sử cao huyết áp, tăng cholestérol huyết, bệnh đái đường loại 1, và lạm dụng rượu, đươc con gái đưa đến phòng cấp cứu. Cô gái nói rằng mẹ cô có những hành động kỳ cục trong giờ vừa qua. Cô nói rằng bệnh nhân không biết bà ở đâu mặc dầu đang ở trong nhà bà. Mẹ cô cũng không nhận biết gia đình mình và nói những lời khó hiểu. HA là 150/80 mmHg, nhịp tim là 90 đập mỗi phút, nhiệt độ 98,9 độ F, và tần số hô hấp là 16 thở mỗi phút. Lúc khám vật lý, bệnh nhân vã mồ hôi, kích động, và run rẩy.Điện tâm đồ cho thấy nhịp xoang với những đoạn ST và sóng T bình thường. Biện pháp nào là thích hợp nhất đối với bệnh nhân này ?
a. Cho ngay một benzodiazepine để điều trị hội chứng cai rượu (ethanol withdrawal)
b. Khởi động stroke team và đưa bệnh nhân trực tiếp đi chụp cắt lớp vi tính
c. Đo ngay đường huyết ở đầu ngón tay (a stat fingerstick) và cho dextrose nếu đường huyết của bệnh nhân thấp.
d. Yêu cầu hội chẩn tâm thần đối với sundowning
e. Cho haloperidol để an thần.
Đáp :
– Câu trả lời đúng là c. Bệnh nhân đã không bao giờ được đo đường huyết bằng chích đầu ngón tay (fingerstick) ở phòng chọn lọc bệnh nhân (triage). Hạ đường huyết có thể giống với một tai biến mạch máu não hay co giật. Do đó điều quan trọng là tất cả các bệnh nhân có trạng thái tâm thần bị biến đổi cần được đo đường huyết với fingerstick. Nồng độ đường huyết nên được xem là một dấu hiệu sinh tồn (vital sign). Hạ đường huyết là một vấn đề thường gặp ở những bệnh nhân với bệnh đái đường loại I. Bệnh cảnh lâm sàng của hạ đường huyết được gây nên bởi sự tiết gia tăng epinephrine, cũng như loạn năng hệ thần kinh trung ương. Những triệu chứng gồm có vã mồ hôi, sự bồn chồn, run, tim nhịp nhanh, đói, và những triệu chứng thần kinh biến thiên từ co giật và hành vi kỳ lạ đến co giật và hôn mê.
– (a) cai rượu (ethanol withdrawal) có thể có bệnh cảnh tương tự hạ đường huyết vì cả hai gồm có những triệu chứng của một tình trạng adrenergic (tim nhịp nhanh, cao huyết áp, và mồ hôi, kích động). Ngay cả khi anh nghi ngờ hội chứng cai rượu, cũng bắt buộc phải kiểm tra đường bằng chích đầu ngón tay.
– (b) Kíp đột qụy (stroke team) phải được khởi động ở những bệnh nhân có những dấu hiệu và triệu chứng của một đột qụy không gây nên bởi hạ đường huyết. Do đó những bệnh nhân này cần được đo đường huyết bằng chọc đầu ngón tay.
– (d) Sundowning để chỉ những người càng lúc càng lú lẫn vào cuối ngày và vào đêm. Sundowning không phải là một căn bệnh, nhưng là một triệu chứng thường xảy ra ở những người bị sa sút trí tuệ (dementia), như bệnh Alzheimer. Nó thường được quan sát ở các khoa phòng bệnh viện hơn là ở phòng cấp cứu.
– (e) Haloperidol thường được sử dụng như một thuốc an thần đối với những bệnh nhân kích động. Tuy nhiên bệnh nhân này kích động do một nguyên nhân thực thể, hạ đường huyết. Bằng cách điều trị nguyên nhân (cho đường), tình trạng kích động sẽ biến mất.
Emergency Medicine : PreTest
Đọc thêm :
Cấp cứu ngộ độc số 21
Cấp cứu nội thần kinh số 25
BS NGUYỄN VĂN THỊNH
(12/4/2015)

Cấp cứu nội thần kinh số 36 – BS Nguyễn Văn Thịnh

(BEWARE OF THE COMORBIDITIES AND COMPLICATIONS OF ACUTE STROKE)
Thomas M. Mailhot, MD
Đột qụy là nguyên nhân dẫn đầu của tỷ lệ bệnh tật và tử vong ở Hoa Kỳ. Khi các bệnh nhân đến phòng cấp cứu với những triệu chứng giống đột qụy (strokelike symptôms), thầy thuốc cấp cứu đứng trước một quyết định khó khăn : cho tPA (tissue Plasminogen activator) hay không cho tPA. Quyết định này là quan trọng, nhưng có nhiều can thiệp khác có thể cũng quan trọng trong việc phòng ngừa tỷ lệ bệnh và tử vong.
Stroke unit care đã được bắt đầu năm 1990 để cải thiện những tiên lượng, mang lại một kíp chuyên môn nhiều chuyên khoa tập trung vào điều trị cấp tính và phục hồi chức năng. Theo American Stroke Association, ” tầm quan trọng của những lợi ích của stroke unit care có thể so sánh với tầm quan trọng của tPA tiêm tĩnh mạch.” Ngoài ra, trong khi chỉ một bộ phận nhỏ những bệnh nhân đột qụy đủ tiêu chuẩn để thực hiện điều trị tiêu huyết khối (fibrinolytics), tất cả những bệnh nhân đột qụy được điều trị chuyên môn của một stroke unit.
Khi những bệnh nhân lưu lại trong thời gian lâu hơn ở phòng cấp cứu để chờ đợi giường nội trú, các thầy thuốc cấp cứu phải đương đầu với nhu cầu bắt đầu những yếu tố của điều trị đột qụy cấp tính mà trước đây được dành cho khoa nhận bệnh. Bắt đầu những điều trị này ở phòng cấp cứu có thể ngăn ngừa tỷ lệ bệnh và tử vong, cũng như rút ngắn thời gian nhập viện. Săn sóc hỗ trợ và điều trị những biến chứng cấp tính trong xử trí đột qụy bao gồm phòng ngừa, nhận biết, và điều trị giảm oxy mô, nhồi máu cơ tim, huyết khối tĩnh mạch sâu, nghẽn tắc động mạch phổi, những nhiễm trùng như nhiễm trùng đường tiểu và viêm phổi do hít dịch (aspiration pneumonia), mất nước, kém dinh dưỡng, thương tổn da, và những rối loạn chuyển hóa.
Phòng ngừa giảm oxy mô là cơ sở của hồi sức thần kinh. Những nạn nhân đột qụy thường có phản xạ đường khí bị giảm do liệt hành não (bulbar palsies) cũng như suy giảm mức độ tri giác. Những bệnh nhân như thế đòi hỏi nội thông khí quản để đảm bảo an toàn đường dẫn khí và phòng ngừa những biến chứng của viêm phổi do hít dịch (aspiration pneumonia). Ngay cả khi không nội thông, những biện pháp để phòng ngừa viêm phổi do hít dịch nên được thực hiện sớm trong điều trị bệnh nhân đột qụy : nâng cao đầu giường lên 30 độ và giữ bệnh nhân nil per os cho đến khi một sự đánh giá nói và nuốt được thực hiện. Oxy cho bổ sung có thể cần để đảm bảo oxygenation não thích đáng, đặc biệt trong khung cảnh giảm thông khí và xẹp phổi. Tuy nhiên liệu pháp oxy không phải là không có độc tính của nó, đặc biệt với những nồng độ oxy kéo dài trên 50%, vì vậy oxy bổ sung nên được chuẩn độ để sử dụng lượng tối thiểu cần thiết. Những guideline hiện nay gợi ý một độ bảo hòa oxy đích ít nhất 92% ở những bệnh nhân đột qụy.
Tăng thân nhiệt (hyperthermia) luôn luôn được liên kết với những tiên lượng xấu trong đột qụy. Sự hiện diện của sốt buộc phải tích cực tìm kiếm một nguyên nhân, như viêm phổi, nhiễm trùng đường tiểu, hay viêm nội tâm mạc. Viêm phổi (thường do hít dịch) là nguyên nhân dẫn đầu gây tử vong ở những bệnh nhân đột qụy. Bất cứ nhiễm trùng nào cũng phải được điều trị một cách nhanh chóng và sốt cần được điều trị bằng thuốc hạ nhiệt và những biện pháp làm lạnh khác nếu cần.
Những nạn nhân đột qụy có nguy cơ cao phát triển cả huyết khối tĩnh mạch sâu (DVT) lẫn decubitus ulcers do hạn chế vận động. Phòng ngừa huyết khối tĩnh mạch sâu có thể được thực hiện bằng cách sử dụng những thiết bị đè ép luân phiên (sequential compression device) trên các cẳng chân. Heparin cần phải tránh trong giai đoạn cấp tính do nguy cơ biến đổi xuất huyết của đột qụy. Decubitus ulcer thường không được thầy thuốc cấp cứu xét đến, nhưng những vết loét này là một mối quan tâm gia tăng khi bệnh nhân lưu lại trong khoa cấp cứu với những khoảng thời gian lâu hơn. Hầu hết những brancard ở phòng cấp cứu có những nệm mỏng không dùng để phòng ngừa pressure ulcer. Ngoài ra, mặc dầu nursing guidelines khuyến nghị thay đổi tư thế bệnh nhân mỗi 2 giờ, nhưng điều này thường không thể thực hiện trong một khoa cấp cứu bận rộn. Dầu cho những trở ngại này, bất cứ cố gắng nào v ề phía nhân viên khoa cấp cứu để chăm lo phòng ngừa loét có thể dẫn đến những tiên lượng bệnh nhân tốt hơn. Đặc biệt, những vùng nhạy cảm đối với vỡ da ở những bệnh nhân nằm gồm có vùng xương cùng và các gót chân.
Kiểm soát glucose là một thành phần quan trọng của điều trị đột qụy. Hạ đường huyết có thể gây những dấu hiệu thần kinh cục bộ giống với đột qụy, do đó tất cả những bệnh nhân được giả định là đột qụy phải được đo glucose huyết một cách nhanh chóng lúc đến phòng cấp cứu. Hạ đường huyết cũng có thể trực tiếp dẫn đến thương tổn não, và ở những bệnh nhân đột qụy bị hạ đường huyết, điều này là một đe dọa thật sự đối với sự hồi phục. Đo glucose huyết thường xuyên sẽ nhận diện hạ đường huyết nhẹ trước khi nó trở thành một vấn đề. Hạ đường huyết cũng được liên kết với tỷ lệ tử vong cao hơn trong đột qụy. Nồng độ đường huyết trên 200 mg/dL phải được điều trị với insulin.
Cao huyết áp là một dấu hiệu thường gặp ở những bệnh nhân đột qụy, và có sự tranh luận về điều trị nó trong bối cảnh cấp tính. Mặc dầu những nạn nhân đột qụy với huyết áp tăng cao nghiêm trọng có những tiên lượng xấu hơn những bệnh nhân có huyết áp bình thường, nhưng không có lợi ích lâm sàng nào được nhận thấy ở những bệnh nhân được điều trị cao huyết áp trong bối cảnh cấp tính. Hạ huyết áp cũng được liên kết với những tiên lượng xấu hơn trong đột qụy và cần phải tránh.
Vì các bệnh nhân đợi càng ngày càng lâu hơn để được đưa vào khoa phòng hay khoa hồi sức, nên gánh nặng của điều trị đột qụy cấp tính chuyển vào khoa cấp cứu. Với sự chú ý những khía cạnh điều trị tuy nhỏ nhưng quan trọng này, thầy thuốc cấp cứu có thể làm giảm tỷ lệ bệnh và tử vong của đột qụy và làm giảm thời gian lưu viện.
Reference : Avoiding Common Errors in the Emergency Department.2010
BS NGUYỄN VĂN THỊNH
(11/4/2015)

Cấp cứu ngoại thần kinh số 7 – BS Nguyễn Văn Thịnh

CẤP CỨU CHẨN ĐOÁN XUẤT HUYẾT DƯỚI MÀNG NHỆN
(HEMORRAGIE SOUS-ARACHNOIDIENNE)

O.L Alves
B. George
Service de neurochirurgie
Hopital Lariboisière (Paris)
Xuất huyết dưới màng nhện (hémorragie sous-arachnoidienne) được định nghĩa như là sự hiện diện của máu trong khoang dưới nhện. Thuật ngữ xuất huyết màng não (hémorragie méningée) để định đặc điểm loại xuất huyết này phải được bỏ, bởi vì thiếu chính xác.
Loại xuất huyết này có thể là hậu quả của một chấn thương sọ hoặc thứ phát một xuất huyết trong nhu mô hay trong não thất. Xuất huyết dưới nhện do sự chảy máu của một huyết quản nằm trong khoang dưới nhện được gọi là ” xuất huyết dưới nhện nguyên phát” hay “ngẫu nhiên”. Trong 70 -75% các trường hợp xuất huyết này tương ứng với sự vỡ của một phình mạch và trong khoảng 5% những trường hợp do một dị dạng động-tĩnh mạch (MAV : malformation artério-veineuse) (Bảng 1)Do mức độ nghiêm trọng và tỷ lệ lưu hành của xuất huyết dưới nhện do vỡ phình mạch, chính chủ yếu dạng này sẽ được trình bày trong bài này. Tỷ lệ xuất huyết dưới nhện theo vị trí và kích thước của phình mạch được chỉ trong bảng II.
Bảng II Tỷ lệ xuất huyết dưới nhện tùy theo kích thước và định vị của phình mạch
I. DỊCH TỂ HỌC
Xuất huyết dưới nhện là một bệnh trầm trọng được liên kết với một tỷ lệ bệnh và tử vong cao. Xuất huyết dưới nhện chiếm từ 6 đến 10% của tất cả những tai biến huyết quản và 25% tỷ lệ tử vong mạch máu não. Tỷ lệ mắc bệnh được ước tính 28 trường hợp đối với 100.000 người mỗi năm. Cao điểm tỷ lệ mắc bệnh nằm giữa 55 và 60 tuổi với một sự gia tăng theo mùa (mùa xuân và mùa thu). Xuất huyết dưới nhện xảy ra ở các phụ nữ nhiều hơn đàn ông. Trong 30 % các trường hợp, xuất huyết dưới nhện xảy ra trong lúc ngủ. Khoảng 50% các bệnh nhân có những dấu hiệu lâm sàng, thường nhất là đau đầu, từ 6 đến 20 ngày trước khi xuất huyết. Nhiều facteur de morbidité của xuất huyết dưới nhện đã được nhận diện : cao huyết áp, chứng nghiện thuốc lá và thiếu hụt collagène loại 3. Các oestroprogestatifs được quy kết ; tuy nhiên cho đến nay, những công trình nghiên cứu lâm sàng được thực hiện không cho phép xác nhận điều đó. Sự ngừng không dùng thuốc ngừa thai bằng đường miệng không được khuyến nghị.
Khoảng 1/3 những bệnh nhân bị xuất huyết dưới nhện chết trước khi được điều trị nội-ngoại khoa, 1/3 có những di chứng thần kinh nghiêm trọng mặc dầu một điều trị thích ứng và chỉ 1/3 tìm lại trạng thái thần kinh truớc khi xuất huyết. Sự xuất huyết khởi đầu và sự tái xuất huyết sớm góp phần một cách quyết định vào tỷ lệ tử vong và vào những di chứng thần kinh nghiêm trọng. Nguy cơ tái xuất huyết là 4% ngày đầu, 20% 6 tuần đầu và 50% 6 tháng đầu tiếp theo sau xuất huyết ban đầu. Sau đó nguy cơ tái xuất huyết ổn định ở mức 3% mỗi năm. Inagawa và cộng sự viên đã cho thấy trên 33 bệnh nhân rằng cao điểm tái phát xuất huyết xảy ra trong 6 giờ đầu sau xuất huyết ban đầu. Điều đó chứng tỏ lợi ích của một chẩn đoán và một điều trị sớm xuất huyết dưới nhện, nhất là tỷ lệ tử vong quan trọng hơn nhiều khi tái chảy máu (70%) so với khi xuất huyết ban đầu.
Ngoài ra, nếu điều trị thỏa mãn trên quan điểm kỹ thuật, nó mang lại cho mỗi bệnh nhân một sự bảo vệ cá nhân tuyệt đối và vĩnh viễn. Trên những séries chirurgicales lớn những phình mạch được mổ, tỷ lệ tái xuất huyết hậu phẫu là khoảng 1% (những phình mạch clipé không hoàn toàn). Ngoài sự tái xuất huyết, xuất huyết dưới nhện có thể kèm theo những biến chứng thần kinh nghiêm trọng khác, gồm có co thắt mạch (vasospasme) (30%), tràn dịch não (hydrocéphalie) (15%), xuất huyết trong nhu mô não (20 đến 30%), xuất huyết trong não thất (13 đến 28%) và máu tụ dưới màng cứng (từ 2 đến 5%)
II. SINH BỆNH LÝ
Trong thời kỳ sau xuất huyết dưới nhện, áp lực nội sọ đặt những mức đáng kể, cho đến những trị số gần với huyết áp trương tâm. Sự gia tăng áp lực nội sọ này gây nên một sự giảm rõ rệt của lưu lượng máu não, đôi khi đi đến ngừng tuần hoàn não. Sự phục hồi dần dần một lưu lượng não, với sự xuất hiện của một sung huyết phản ứng (hyperémie réactive), xác định mức độ lan rộng của những thường tổn não ở mức tế bào và, do đó, tiên lượng thần kinh. Ngoài ra, trong xuất huyết dưới nhện, được mô tả những bất thường của điện tâm đồ (những rối loạn khử cực và của nhịp tim có thể tạo nên một bệnh cảnh suy tim với phù phổi cấp), được giải thích bởi kích thích giao cảm mạnh nguồn gốc trung ương, với tăng tiết noradrénaline, mà điểm khởi đầu là thương tổn vùng dưới đồi (hypothalamus).
III. CHẨN ĐOÁN
Thường nhất, chẩn đoán đã rõ ràng. Trong thể điển hình của xuất huyết dưới nhện, các bệnh nhân có một đau đầu thường được mô tả như là dữ dội nhất trong đời họ. Hai yếu tố rất gợi đến một xuất huyết dưới nhện : khởi đầu đột ngột và sự xuất hiện trong khi gắng sức vật lý. Một hội chứng màng não (syndrome méningé) (sự hiện diện của cứng gáy, các dấu hiệu Kernig và Brudzinski) liên kết với nôn mửa và triệu chứng sợ ánh sáng (photophobie) nói chung đi kèm đau đầu. Sự xuất hiện của một thiếu sót vận động (déficit moteur), một mất ngôn ngữ (aphasie) hay một biến đổi của tri giác khiến nghĩ rằng có một máu tụ liên kết trong não. Các dạng lâm sàng không điển hình cũng phải được ghi nhớ : hội chứng Terson (hémorragie intravitréenne), các hội chứng lú lẫn (syndrome confusionnel) hay mutisme akinétique do vỡ một phình mạch của động mạch thông trước (artère communicante antérieure). Sự xuất hiện đột ngột của một liệt hoàn toàn của dây thần kinh sọ số III gợi ý một sự bành trướng của một phình mạch của động mạch thông sau, cần một điều trị cấp cứu.
Khi chẩn đoán xuất huyết dưới nhện được nghi ngờ, một chụp cắt lớp vi tính não không tiêm thuốc cản quang được đòi hỏi trước hết. Được thực hiện trong 24 giờ đầu sau khi xuất huyết, nó cho phép phát hiện xuất huyết trong 95% các trường hợp. Chụp cắt lớp vi tính não ban đầu có một giá trị tiên lượng quan trọng. Thật vậy, sự xuất hiện một co thắt mạch (vasospasme) rất tương quan với vị trí và thể tích của xuất huyết được thấy trên chụp cắt lớp vi tính (Grade de Fisher). Ngoài ra chụp cắt lớp vi tính sẽ cho phép phát hiện những biến chứng nội sọ khác như tràn dịch não (hydrocéphalie). Sự định vị ưu tiên của những cục máu đông trong một trong các bể (citerne) gợi ý vị trí của thương tổn phình mạch trong 70% các trường hợp. Trong trường hợp nhiều phình mạch, như thế chụp cắt lớp vi tính cho phép nhận diện phình mạch chịu trách nhiệm xuất huyết nhờ quan sát épicentre của xuất huyết.
Khi chụp cắt lớp vi tính não không phát hiện xuất huyết và trước một bệnh cảnh lâm sàng rất đáng nghi ngờ, cần thực hiện chọc dò dịch não tủy để chứng minh sự hiện diện của máu mới xảy ra. Sự hiện diện của một dịch não tủy nhiễm sắc tố vàng (xanthochromatique) phân biệt xuất huyết dưới nhện với xuất huyết do chọc dò. Khoảng 70% các dịch não tủy cho thấy một dịch não tủy nhiễm sắc tố vàng 6 giờ sau khi xuất huyết. Như thế cần khuyến nghị trì hoãn (hay lập lại) chọc dò dịch não tủy cho đến giờ thứ 12 sau khi bắt đầu một triệu chứng đau đầu riêng rẽ, bởi vì chỉ vào lúc đó mà chọc dò mới dương tính.
Chụp mạch máu não (angiographie cérébrale) 4 trục phải được thực hiện tức thời với mục tiêu làm cho nhìn thấy phình mạch và xác định những đặc điểm cấu trúc mạch máu của nó. Thăm dò này sẽ xác nhận vị trí, kích thước, các đặc điểm của cổ (collet) phình mạch và sự hiện diện của những phình mạch liên kết. Tất cả tham số này góp phần để chọn lựa điều trị thích ứng nhất, ngoại khoa hay nội mạch (endovasculaire). Tỷ lệ biến chứng trong khi thực hiện một chụp mạch máu não chẩn đoán (angiographie cérébrale diagnostique) là không đáng kể, dưới 1%. trong 15 đến 20% các trường hợp, chụp động mạch không cho phép xác nhận dị tật mạch máu (malformation vasculaire). Khi đó cần chụp lại mạch máu vào ngày thứ hai. Khi xuất huyết dưới nhện có một phân bố quanh thân não lúc làm scanner, sự thực hiện một chụp mạch máu não thứ hai không được chỉ định. Thật vậy, loại xuất huyết dưới nhện này là một thực thể lâm sàng riêng ; nói chung nó liên kết với một tiến triển lâm sàng thuận lợi và không bị những biến chứng thường xảy ra. Nguyên nhân của sự xuất huyết thường vẫn không được giải thích.
Chụp động mạch được xem như thăm dò tối ưu để chẩn đoán bệnh phình mạch. Ngoài những chi tiết cơ thể học của phình mạch, chụp động mạch cung cấp một thông tin quý về tuần hoàn bàng hệ của polygone de Willis và về sự hiện diện của một co thắt mạch máu (vasospasme). Tuy nhiên, những thăm dò khác, như angio-IRM (chụp mạch cộng hưởng từ) hay angio-scanner (chụp mạch cắt lớp vi tính) sẽ cung cấp trong thời gian ngắn, nhờ một sự cải thiện của logiciel informatique, những thông tin cũng chính xác như chụp động mạch. Thật vậy, angio-IRM (chụp mạch cộng hưởng từ) hiện nay có một mức độ nhạy cảm 86% trong việc phát hiện phình mạch có kích thước trên 3 mm. Hiệu năng của thăm dò này bị hạn chế bởi kích thước của phình mạch, lượng và chiều của lượng máu trong túi của phình mạch so với từ trường, cũng như sự hiện diện của huyết khối hay vôi hóa trong phình mạch. Ngược lại, chụp mạch cắt lớp vi tính (angioscanner) có một độ nhạy cảm 95% và một độ đặc hiệu 83% trong chẩn đoán những phình mạch có kích thước trên 2,2 mm. Nó cho phép thấy phình mạch theo ba chiều và xác định những tương quan cơ thể học của nó với những cấu trúc xương lân cận, điều này có tầm quan trọng chủ yếu đối với phẫu thuật viên. Hiện nay hai thăm dò không xâm nhập này được ưu tiên dành có phát hiện khi có một nghi ngờ chẩn đoán lớn. Đó là trường hợp những thể gia đình trong đó hơn hai người mang một phình mạch.
Reference : Urgences neurologiques
Đọc thêm : Cấp cứu ngoại thần kinh số 1, 2, 5
BS NGUYỄN VĂN THỊNH
(7/4/2015)

Cấp cứu Nội Tiết + Chuyển Hoá số 23 – BS Nguyễn Văn Thịnh

ĐỪNG CHẨN ĐOÁN MỘT BỆNH NHÂN HẠ ĐƯỜNG HUYẾT BẰNG CHỤP CẮT LỚP VI TÍNH
(DO NOT FIND OUT THAT YOUR PATIENT IS HYPOGLYCEMIC ON THE CT SCANNER)

Zhanna Livshits, MD
Department of Emergency Medicine
New York University School of Medicine
Andrew Stolbach, M.D
Assistant Professor
Department of Emergency Medicine
Johns Hopkins University & Hospital
Baltimore, Maryland
Hạ đường huyết dễ chẩn đoán…nếu như anh nghĩ đến nó. Bệnh nhân hạ đường huyết “cổ điển” có những triệu chứng của hệ thần kinh tự trị (autonomic symptoms), và của giảm glycogène thần kinh (neuroglycopenic symptoms). Những triệu chứng của hệ thần kinh tự trị, kết quả của sự phóng thích catecholamine, gồm có run rẩy (tremor), tim nhịp nhanh và ra mồ hôi. Những triệu chứng giảm glycogène thần kinh (neuroglycopenic symptoms), do tác dụng của mất glucose (glucose deprivation) ở não, gồm có lú lẫn, co giật, và hôn mê. Bất hạnh thay, nhiều bệnh nhân không biểu hiện một cách “cổ điển”. Hạ đường huyết nên được xét đến ở những bệnh nhân với bất cứ bất thường tâm-thần kinh nào, bao gồm những thiếu sót thần kinh cục bộ (focal neurologic deficits).
Vài bệnh nhân không nhận biết tình trạng hạ đường huyết của mình. Những bệnh nhân đái đường được kiểm soát tốt ít có khả năng hơn những bệnh nhân khác nhận biết những triệu chứng khởi đầu của hạ đường huyết. Những bệnh nhân dùng beta-blocker có thể không có những triệu chứng tự trị đáng kể, làm chẩn đoán không rõ ràng đối với nhà lâm sàng cũng như bệnh nhân.
Ở vài quần thể bệnh nhân, các nhà lâm sàng có thể quên không xét đến hạ đường huyết. Hãy đo nồng độ đường huyết ở bệnh nhân ngộ độc nằm trên brancard trong hành lang. Những bệnh nhân nghiện rượu đặc biệt dễ bị hạ đường huyết, bởi vì ethanol ức chế sự tân sinh đường (gluconeogenesis) và làm vơi những dự trữ glycogen. Bởi vì hạ đường huyết có thể gây nên một thiếu sót thần kinh khu trú, vậy hãy kiểm tra đường huyết trước khi bạn gởi chụp cắt lớp vi tính bệnh nhân bại liệt nửa người của bạn ” để loại trừ tai biến mạch máu não”
Một khi bạn đã có được số đo đường huyết, hãy cẩn thận khi giải thích nó. Hạ đường huyết là một chẩn đoán vừa lâm sàng vừa xét nghiệm.Trong một công trình nghiên cứu, những bệnh nhân đái đường kiểm soát kém đầu tiên phát những triệu chứng ở một nồng độ đường huyết trung bình 78 mg/dL, trong khi những người không bị đái đường trở nên có triệu chứng ở một nồng độ đường huyết trung bình 53 mg/dL. Bệnh đái đường của bạn với một con số đường huyết “bình thường” có thể đúng là bị hạ đường huyết.
Vài thầy thuốc không điều trị đủ bệnh nhân bị hạ đường huyết của họ. Điều trị đối với hạ đường huyết là cho dextrose bằng đường tĩnh mạch, tiếp theo sau bởi một bữa ăn. Nên cho bệnh nhân trưởng thành 0,5 đến 1g/kg D50W (50% concentrated dextrose) bằng đường tĩnh mạch, ở trẻ em D25W (25% concentrated dextrose solution), và ở nhũ nhi D10W. Bởi vì “amp” chuẩn có 25g dextrose, hầu như tất cả những bệnh nhân người lớn phải nhận hơn một seringe. Mỗi trong những “amps” này chỉ có 10 cal. Bolus khởi đầu nên được tiếp theo bởi một bữa ăn. Nếu bệnh nhân có thể ăn, hãy cho bệnh nhân ăn. Một bữa ăn sẽ cung cấp nhiều giờ những caloris đường phóng thích kéo dài.
Tiêm truyền dextrose có thể cần thiết đối với những bệnh nhân không thể ăn được hay bị quá liều một dạng insulin có tác dụng kéo dài. Khi ra lệnh tiêm truyền, phải nhớ rằng D5 không phải là dung dịch thay thế glucose. Mỗi túi 1 L D5 chứa 50 g glucose, hay 200 cal. Cho dung dịch này với tốc độ 100 mL mỗi giờ sẽ cấp một lượng không đáng kể 20 cal mỗi giờ cho bệnh nhân. Một lựa chọn tốt hơn là chuẩn bị một dung dịch dextrose 10% và cho bổ sung với D50. Một cách khác là cho dextrose 20%, nhưng phải được cho qua một catheter tĩnh mạch trung tâm để tránh độc tính đối với tĩnh mạch.
Glucagon thường được tiêm mông cho những bệnh nhân hạ đường huyết không có đường tĩnh mạch. Glucagon đòi hỏi dự trữ glycogen thích đáng để hiệu quả và do đó có thể không hiệu quả ở trẻ em, người già, và ở những bệnh nhân với dự trữ chuyển hóa giảm do nghiện rượu hay bệnh mãn tính. Bởi vì glucagon chỉ là biện pháp trì hoãn và không mang lại lợi ịch hơn dextrose đơn độc, nên không có vai trò đối với glucagon để xử trí hạ đường huyết đối với những bệnh nhân với đường tĩnh mạch
Những bệnh nhân sử dụng insulin có thời gian tác dụng ngắn có thể được cho xuất viện nếu họ không đòi hỏi điều trị thêm đối với hạ đường huyết trong thời gian quan sát 4 hay 6 giờ. Bởi vì kinetics của insulin không thể tiên đoán được trong overdose, nên những bệnh nhân với intentional overdose nên được nhập viện. Trong một trường hợp lente insulin overdose, hạ đường huyết không xảy ra cho đến 18 giờ sau overdose. Một cách tương tự, những bệnh nhân với suy thận, suy gan, hay hạ đường huyết không giải thích được cần phải nhập viện.
Take home point là gì ? Anh không thể chẩn đoán hạ đường huyết nếu anh không nghĩ đến nó. Hạ đường huyết có thể được biểu hiện với bất cứ bất thường thần kinh-tâm thần nào và có thể hiện diện với một đường huyết ” bình thường “. Một khi anh xác định là hạ đường huyết, điều trị thích hơp không chỉ một “amp” D50 và cho về nhà. Bệnh nhân phải được cho ăn một bữa ăn, và cần được nhập viện những bệnh nhân được chọn lọc. Nếu anh không xác định được nguyên nhân của hạ đường huyết, bệnh nhân ở lại bệnh viện an toàn hơn.
Reference : Avoiding Common Errors in the Emergency Department
BS NGUYỄN VĂN THỊNH
(7/4/2015)

Cấp cứu hô hấp số 35 – BS Nguyễn Văn Thịnh

BỆNH LAO
(TUBERCULOSIS)

Amy J. Behrman, M.D
Associate Professor of Emergency Medicine
University of Pensylvania Health System
Philadelphia, Pennsylvania.
I. NHỮNG ĐẶC ĐIỂM LÂM SÀNG
1/LAO NGUYÊN PHÁT (PRIMARY TUBERCULOSIS) :
Nhiễm lao nguyên phát thường không có triệu chứng, thường nhất được thể hiện bởi phản ứng dương tính đối với trắc nghiệm da. Tuy nhiên vài bệnh nhân có thể có viêm phổi hoạt động hay bệnh ngoài phổi. Những bệnh nhân bị suy giảm miễn dịch có khả năng hơn phát triển những nhiễm lao nguyên phát tiến triển nhanh.
2/ LAO TÁI HOẠT (REACTIVATION TUBERCULOSIS)
Những nhiễm lao tiềm tàng (latent tuberculosis infection) không có triệu chứng với các trắc nghiệm lao tố da (tuberculin skin test) dương tính. Những nhiễm lao tiềm tàng sẽ tiến triển thành bênh hoạt động trong 5% các trường hợp trong vòng 2 năm sau khi nhiễm lao nguyên phát ; thêm 5% sẽ tái hoạt trong suốt cuộc đời. Tỷ lệ tái hoạt cao hơn ở người trẻ, người già, những người với nhiễm lao nguyên phát mới đây, những người với suy giảm miễn dịch (đặc biệt HIV), và những người với những bệnh mãn tính như đái đường và suy thận. Hầu hết các bệnh nhân biểu hiện bán cấp tính với sốt, ho, mất cân, mệt và ra mồ hôi đêm (night sweats).
Hầu hết những bệnh nhân với bệnh lao hoạt động (active tuberculosis) có thương tổn phổi, được đặc trưng bởi những triệu chứng toàn thân và ho (thường có đờm). Ho ra máu, pleuritic chest pain, và khó thở có thể phát triển. Ran và ronchi có thể được tìm thấy, nhưng khám phổi thường không chẩn đoán.
Lao ngoài phổi (extra-pulmonary TB) phát triển trong 15% các trường hợp. Viêm hạch bạch huyết (lymphadenitis), với hạch lớn và không đau và draining sinuses là những biểu hiện thường gặp nhất. Tràn dịch màng phổi triệu chứng, viêm ngoại tâm mạc, viêm màng bụng, hay viêm màng não có thể là biểu hiện của bệnh lao. Tiến triển thường cấp tính hơn ở trẻ em.
Lao kê là một bệnh nhiều hệ (a multisystem disease) được gây nên bởi sự phân tán ồ ạt bằng đường máu. Lao kê thường gặp nhất ở những bệnh nhân bị suy giảm miễn dịch và các trẻ em. Những triệu chứng và những dấu hiệu gồm có sốt, ho, mất cân, nổi hạch, to gan và lách, và cytopenia. Lao ngoài phổi cũng xảy ra ở xương, khớp, da, thận và các tuyến thượng thận.
HIV VÀ LAO
Những bệnh nhân bị suy giảm miễn dịch và đặc biệt những bệnh nhân HIV vô cùng nhạy cảm với bệnh lao và có khả nặng hơn nhiều phát những nhiễm trùng hoạt động với những bệnh cảnh không điển hình. Lao phổi nên được xét đến ở bất cứ bệnh nhân HIV nào với những triệu chứng hô hấp, mặc dầu những phim X quang ngực bình thường. Lao phân tán ngoài phổi cũng thường gặp ở những bệnh nhân HIV và nên được xét đến trong đánh giá những triệu chứng ngoài phổi.
LAO ĐỀ KHÁNG VỚI NHIỀU THUỐC
MDR-TB (Multidrug-resistant TB) đạt cao điểm trong sự trổi dậy mới đây của bệnh lao trong đất nước này, đặc biệt trong số những bệnh nhân HIV. Bệnh lao vẫn là một vấn đề lớn trong số những người sinh ở nước ngoài, chịu trách nhiệm 72% những trường hợp MDR-TB năm 2000. Điều này nên được xét đến ở bất cứ nơi nào benh lao được chẩn đoán, đặc biệt trong số những người với điều trị dưới tối ưu trước đây như những người di cư đến từ những vùng dịch bệnh, những tù nhân, những người vô gia cư, và những người nghiện ma túy. MDR-TB vẫn còn thường gặp ở những bệnh nhân HIV hơn là dân thường và có một tỷ lệ tử vong cao trong nhóm này.
II/ CHẨN ĐOÁN VÀ CHẨN ĐOÁN PHÂN BIỆT
Hãy xét chẩn đoán lao phổi ở bất cứ bệnh nhân nào với những triệu chứng hô hấp hay toàn thân để chẩn đoán sớm, bảo vệ nhân viên bệnh viện, và xử trí thích hợp.
Chụp X quang ngực là công cụ chẩn đoán hữu ích nhất đối với lao phổi hoạt động (active TB) ở phòng cấp cứu. Bệnh lao phổi nguyên phát hoạt động (active primary tuberculosis) thường được biểu hiện bởi những thâm nhiễm nhu mô ở bất cứ vùng phổi nào. Hạch vùng rốn phổi và/hoặc trung thất có thể xảy ra có hay không có thâm nhiễm. Những thương tổn có thể vôi hóa. Lao tái hoạt (reactivation TB) được biểu hiện bởi những thương tổn ở các thùy trên hay những phân thùy trên của thùy dưới. Có thể thấy cavitation, vôi hóa, sẹo hóa, xẹp phổi, và tràn dịch màng phổi. Cavitation có thể liên kết với khả năng gây nhiễm gia tăng. Lao kê có thể gây những thâm nhiễm nốt (nodular infiltrates) nhỏ (1-3mm), phân tán. Những bệnh nhân đồng nhiễm HIV và TB đặc biệt có thể có những phim chụp X quang ngực không điển hình.
Nhuộm kháng acid (acid fast staining) đờm có thể phát hiện mycobacteria trong 60% những bệnh nhân với lao phổi, mặc dầu hiệu năng thấp hơn ở những bệnh nhân HIV. Những mycobacterium không điển hình có thể cho những dương tính giả. Nhiều bệnh nhân sẽ có những âm tính giả trên một mẫu nghiệm đờm duy nhất. Soi hiển vi những mẫu nghiệm không phải đờm (dịch màng phổi và dịch não tủy) ít nhạy cảm hơn. Nói chung cấy xác định cần nhiều tuần, nhưng những trắc nghiệm di truyền mới dùng DNA probe hay kỹ thuật PCR (polymerase chain reaction) có thể xác nhận chẩn đoán trong nhiều giờ.
Trắc nghiệm Mantoux (trắc nghiệm da lao tố trong bì với PPD) nhận diện hầu hết những bệnh nhân với nhiễm lao tiềm tàng, trước đây hay đang hoạt động. Những kết quả được đọc 48 đến 72 giờ sau khi đặt, do đó hạn chế sự hữu ích của trắc nghiệm này đối với những bệnh nhân cấp cứu. Những bệnh nhân với HIV hay những tình trạng suy giảm miễn dịch khác và những bệnh nhân với disseminated TB có thể có những trắc nghiệm da âm tính giả ngay cả không mất ứng (anergic) hoàn toàn
III. XỬ TRÍ Ở PHÒNG CẤP CỨU
1. Điều trị ban đầu phải bao gồm ít nhất 4 loại thuốc cho đến khi có profile đối với một bệnh nhân. Điều trị bắt đầu thường gồm có isoniazid (INH), rifampicin, pyrazinamide, và streptomycin hay ethambutol trong hai tháng. Ít nhất hai loại thuốc được tiếp tục thêm 4 tháng nữa. Những bệnh nhân bị suy giảm miễn dịch hay MDR-TB có thể đòi hỏi nhiều loại thuốc hơn trong những thời gian dài hơn. Bảng dưới đây tóm tắt những liều thuốc ban đầu mỗi ngày và những tác dụng phụ. Các thuốc này có thể được thích ứng theo các kết quả cấy và kháng sinh đồ khi được biết.2. Nhập viện được chỉ định đối với tình trạng bất ổn lâm sàng, chẩn đoán không chắc chắn, theo dõi và sự tuân thủ điều trị ngoại trú không đáng tin cậy, và bệnh nhân có bệnh lao hoạt động và đề kháng nhiều loại thuốc được biết. Các thầy thuốc cần biết các luật địa phương về nhập viện và điều trị ngoài ý muốn. Nhập viện để cách ly hô hấp hay “droplet” là bắt buộc đối với tất cả những trường hợp nghi bệnh lao.
3. Những bệnh nhân với bệnh lao hoạt động (active TB) được cho ra khỏi phòng cấp cứu phải chuyển ngay cho một thầy thuốc hay sở y tế địa phương để được điều trị trong thời gian dài hạn. Các bệnh nhân nên được giáo dục về cách ly tại nhà, theo dõi, và screening những người tiếp xúc trong hộ. Có những chế độ thuốc mỗi ngày hay từng đợt. Vài bệnh nhân đòi hỏi điều trị được theo dõi trực tiếp để đảm bảo sự tuân thủ. Những bệnh nhân với PPD dương tính và không có bệnh lao hoạt động cần được đánh giá để điều trị dự phòng với INH để ngăn ngừa bệnh lao tái hoạt (reactivation TB).
4. Nhân viên phòng cấp cứu cần được huấn luyện để nhận diện những bệnh nhân có nguy cơ bị bệnh lao hoạt động càng sớm càng tốt ở phòng cấp cứu và trong tiền viện. Những bệnh nhân nghi lao phổi cần mang masque hay đặt trong những phòng cách ly hô hấp (respiratory isolation room) ở phòng cấp cứu. Các bệnh nhân phải được vận chuyển trong khi đeo masque và được đưa vào những vùng cách ly hô hấp. Nhân viên săn sóc những bệnh nhân nghi bị lao phải mang protection được chấp nhận bởi Occupational Safety and Health Administration. Enginerring controls ở phòng cấp cứu có thể giảm thiểu sự lan truyền từ những trường hợp không được chẩn đoán. Nhân viên phòng cấp cứu phải nhận trắc nghiệm da PPD đều đặn để phát hiện những nhiễm lao nguyên phát mới, loại bỏ bệnh đang hoạt động, và xét đến dự phòng bằng INH
References : Emergency Medicine Manual. Sixth Edition. 2004
BS NGUYỄN VĂN THỊNH
(6/4/2015)

Cấp cứu hô hấp số 34 – BS Nguyễn Văn Thịnh

ĐỪNG LOẠI BỎ BỆNH LAO CHỈ DỰA TRÊN MỘT X QUANG NGỰC ÂM TÍNH
(DO NOT EXCLUDE TUBERCULOSIS SIMPLY BASED ON A “NEGATIVE” CHEST X RAY)

MUSTAPHA SAHEED, MD
Clinical Instructor
Department of Emergency Medicine
Johns Hopkins University & Hospital
Baltimore, Maryland
Lao là một trong những nguyên nhân nhiễm trùng gây tử vong thường gặp nhất trên thế giới. Hơn 8 triệu người phát bệnh lao hoạt động (active tuberculosis), và thêm 2 triệu chết vì bệnh lao mỗi năm. Mặc dầu những tỷ suất nhiễm lao nói chung đã giảm trong các quốc gia công nghiệp, nhưng những kiểu di dân từ những vùng dịch bệnh có thể đang gây nên một sự gia tăng ở vài nơi của thế giới phát triển. Những kiểu đề kháng kháng sinh đang tiến triển và khả năng truyền bệnh tương đối cao làm cho sự nhận diện, sự cách ly (quarantine), và điều trị nhiễm trùng này thành một ưu tiên. Bất hạnh thay, sự nhận diện sớm có thể khá khó vì những triệu chứng của nhiễm trùng lao không đặc hiệu, và xét nghiệm chẩn đoán tương đối phức tạp. Chụp X quang ngực đã là một phần trung tâm của algorithm chẩn đoán. Tuy nhiên, mặc dầu “common misconceptions”, chụp X quang có nhiều hạn chế trong chẩn đoán bệnh lao.
Mycobacterium là một trực khuẩn nội bào với khả năng phòng vệ phát triển tốt chống lại sự miễn dịch của ký chủ. Sau khi tiếp xúc ban đầu với các vi khuẩn trong các giọt nhỏ hô hấp (respiratory droplets), những người có năng lực miễn dịch (immunocompetent) đáp trả bằng một đáp ứng thực bào qua trung gian tế bào (cell-mediated phagocytic response). Bất hạnh thay đáp ứng này nói chung không thành công vì mycobacterium TB có thể sống sót trong tế bào của các đại thực bào. Các vi khuẩn tăng sinh một cách nhanh chóng, xâm nhập các mô tại chỗ và các hạch bạch huyết. Trong 2 đến 4 tuần tiếp theo, ký chủ phản ứng bằng một phản ứng thể dịch được trung gian bởi tế bào T (a T-cell mediated humoral response), một sự kiện được khai thác về mặt chẩn đoán bằng trắc nghiệm da tuberculin. Với sự xuất hiện của tính miễn dịch đặc hiệu, hầu hết những người có năng lực miễn dịch bây giờ sẽ có thể chận đứng sự bành trường của vi khuẩn và nhiên hậu quét sạch trùng. Tuy nhiên sau nhiễm trùng nguyên phát, 5% những người bị tiếp xúc sẽ tiếp tục phát triển một nhiễm trùng nguyên phát tiến triển không ngừng (progressive primary infection). Những người khác sẽ phát triển bệnh lao tiềm tàng (latent TB), được đặc trưng bởi những ổ không nhiễm khuẩn nằm yên ngủ cho đến khi, vì những lý do không rõ ràng, hệ miễn dịch thất bại, và nhiễm trùng tái hoạt động. Trong số những người với bệnh lao nguyên phát, 5 đến 10% sẽ phát triển bệnh lao tái hoạt (reactivation TB).
Trong nhiễm lao nguyên phát (primary TB infection), những dấu hiệu X quang biến thiên. Không như nhiễm lao vùng phổi trên được mô tả một cách cổ điển trong bệnh lao tái hoạt (reactivation TB), lao nguyên phát có khuynh hướng xuất hiện như một quá trình thâm nhiễm (infiltrative process) với một sự ưa thích đối với những thùy dưới. Ngoài ra, 10 đến 20% các bệnh nhân có thể có một thâm nhiễm hang đơn độc (a solitary cavitary infiltrate) hay nhiều thâm nhiễm nốt lớn (macronodular infiltrates), và 10% có thể chỉ có một tràn dịch màng phổi.
Trong trường hợp điển hình bệnh lao tái hoạt (reactivation TB) xảy ra ở vùng trên của phổi. Đáng chú ý là những phân thùy sau và đỉnh (posterior and apical segments) của thùy trên của phổi phải, những phân thùy sau và đỉnh của thùy trên của phổi trái và những phân thùy trên của các thùy dưới bị thương tổn theo thứ tự giảm dần. Khởi đầu, có thương tổn xơ -nốt (fibronodular), tiến triển thành những nốt lớn hơn khi các thương tổn kết hợp lại và thường dẫn đến những thương tổn hang (cavitary lesions) có thể thay đổi kích thước và thường có hoại tử ở vùng trung tâm .
Tuy nhiên, mặc dầu những dấu hiệu X quang được mô tả một cách phân biệt, sự phân biệt giữa nhiễm lao nguyên phát (primary TB) với bệnh lao tái hoạt (reactivation TB) hay nhiễm lao nguyên phát tiến triển không ngừng (progressive primary TB) trên những đặc điểm X quang là không đáng tin cậy. Geng và các công sự viên (2005) trong một retrospective review 456 bệnh nhân có cấy dương tính đối với lao, đã nhận thấy rằng thời gian mắc bệnh và tiến triển lâm sàng không tiên đoán một cách chính xác những bất thường X quang.
Ngoài ra, những bệnh nhân lao thường có những biểu hiện X quang thay đổi, đôi khi không có những đặc điểm phân biệt với những nguyên nhân khác của viêm phổi cộng đồng. Van Cleef và các công sự viên (2005) đã đánh giá 993 bệnh nhân nghi lao dựa trên triệu chứng. Các bệnh nhân được thực hiện 3 kính phết và cấy đờm (sputum smear and culture) và một phim chụp X quang ngực. Điều đáng ghi nhận, 46 trong số 226 bệnh nhân với cấy đờm dương tính có những phim chụp ngực được giải thích như là ” bệnh lý nhưng không phải lao”, hay ” không bệnh lý”, như thế gợi ý một tỷ lệ bỏ sót X quang 20%. Những dữ kiện của họ chứng minh sự khó khăn trong việc loại bỏ chẩn đoán lao dựa trên những dấu hiệu X quang riêng rẻ và nhấn mạnh tính chất biến thiên của hình ảnh X quang.
Ngoài ra, đã có chứng cớ đáng kể cho thấy rằng những dấu hiệu Xquang là chỉ dấu tiên đoán tồi về căn nguyên viêm phổi. Trong một series 162 bệnh nhân với viêm phổi có căn nguyên được biết, được xác nhận bởi cấy đờm, cấy máu, và những xét nghiệm huyết thanh, Boersma và các cộng sự viên (2006) đã không nhận thấy sự liên quan giữa những đặc điểm X quang chung và những tác nhân gây bệnh.
Những hạn chế của X quang rõ ràng không được thay thế bởi sự sử dụng chụp cắt lớp vi tính. Điều chắc chắn, CT phức tạp hơn, cho phép chụp nhu mô phổi theo 3 chiều không gian. Tuy nhiên mức độ nhạy cảm của nó phải được cân bằng với sự cần giảm thiểu điều trị quá đáng (overtreatment), phí tổn lớn hơn, và sự gia tăng tiếp xúc phóng xạ. Trong một công trình nghiên cứu đánh giá những nhiễm trùng phổi ở những bệnh nhân với AIDS, Kang và các cộng sự viên (1996) đã cho thấy rằng CT scan có một độ nhạy cảm gia tăng 96% so với chụp X quang ngực (90%). Các tác giả kết luận rằng sự cải thiện khiêm tốn với chụp hình ảnh bằng chụp cắt lớp vi tính làm nó hữu ích hơn đối với những thăm dò có trọng điểm (targeted testing), trái với thăm dò sàng lọc (screening). Nói chung, chụp cắt lớp vi tính được khuyến nghị đối với những bệnh nhân bị suy giảm miễn dịch hay những bệnh nhân với nghi ngờ cao bị nhiễm trùng bởi vì những bệnh nhân này có khả năng nhiều hơn có những phim X quang không điển hình hay âm tính.
Những thăm dò chụp hình ảnh, đặc biệt chụp X quang ngực chuẩn, vẫn còn là một thành phần chủ yếu của algorithm chẩn đoán đối với bệnh lao. Khi được sử dụng trong bối cảnh lâm sàng thích hợp và cùng với những công cụ chẩn đoán khác gồm trắc nghiệm da lao tố (tuberculin skin test), phim X quang ngực giúp nhận diện những bệnh nhân với nhiễm trùng lao hoạt động. Tuy nhiên, ở những bệnh nhân với một khả năng bị bệnh trước trắc nghiệm từ trung bình đến cao, tính chất biến thiên của những hình ảnh X quang và sự khó tiên đoán những tác nhân gây bệnh từ những dấu hiệu X quang loại trừ việc sử dụng chụp X quang ngực riêng rẻ để loại bỏ chẩn đoán bệnh lao.
Reference : Avoiding common errors in the emergency department. 2010
BS NGUYỄN VĂN THỊNH
(6/4/2015)

Cấp cứu ngoại thần kinh số 6 – BS Nguyễn Văn Thịnh

CHÈN ÉP TỦY CẤP TÍNH
(COMPRESSION MEDULLAIRE AIGUE)

N.Bruder
Service d’Anesthésie-Réanimation
Hôpital de la Timone, Marseille
Chèn ép tủy bán cấp (compression médullaire subaigue) là một cấp cứu ngoại khoa. Sự nặng thêm về mặt lâm sàng, dẫn đến bại liệt hai chi (paraplégie) hay bại liệt bốn chi (tétraplégie), thường rất nhanh và những khả năng phục hồi phần lớn tùy thuộc vào mức độ nhanh chóng của giải phóng chèn ép tủy (décompression médullaire).

I. LÂM SÀNG
Yếu tố chính là sự xuất hiện dần dần của một thiếu sót cảm giác và vận động mà giới hạn trên chỉ mức thương tổn. Triệu chứng học thường bắt đầu bằng những rối loạn bước, rồi một thiếu sót vận động invalidant xuất hiện và kèm theo những rối loạn cơ vòng (troubles sphinctériens). Trong những thương tổn vùng thắt lưng thấp (lésions lombaires basses), triệu chứng học có thể chỉ là một hội chứng đuôi ngựa (syndrome de la queue de cheval). Những dấu hiệu này có thể được đi trước bởi những triệu chứng đau ở vùng lưng hay cổ, đau rễ thần kinh (douleur radiculaire). Đau và yếu cơ có thể kéo dài nhiều tuần nhưng sự trở nặng thành liệt hai chi hay liệt 4 chi có thể xuất hiện trong vài giờ.
II. CĂN NGUYÊN
1. CHẤN THƯƠNG
Khi xảy ra một chấn thương, chẩn đoán hiển nhiên khi hiện hữu tức thời một thiếu sót thần kinh. Thiếu sót này có thể xuất hiện thứ phát trong những thương tổn không vững (lésions instables) không được nhận thấy, nhất là ở cổ, hay khi xuất hiện một thoát vị đĩa đè ép (hernie discale compressive).
2. ĐÈ ÉP DO UNG THƯ (COMPRESSION NEOPLASIQUE)
Đó là nguyên nhân thường gặp nhất. Đè ép tủy xảy ra trong những di căn cột sống của các ung thư (nhất là ung thư phổi, vú, tiền liệt tuyến, thận và tuyến giáp), trong những syndrome myélo-prolifératifs (myélome, lymphome, ung thư bạch cầu).
3. MÁU TỤ NGOÀI MÀNG CỨNG (HEMATOME EPIDURAL)
Phải được gợi lên ở một bệnh nhân đang điều trị kháng đông hay bị những rối loạn đông máu, sau khi chọc dò tủy sống, hay sau khi mổ cột sống (chirurgie rachidienne). Trong nguyên nhân này thường có đau dữ dội.
4. ÁP XE NGOÀI MÀNG CỨNG (ABCES EPIDURAL)
Có thể do là biến chứng của viêm xương tủy (ostéomyélite). Phải gợi lên khi có sốt, nhất là một động tác ngoại khoa hay một chọc dò đã được thực hiện nhiều tháng trước. Triệu chứng học thường được đi trước bởi đau dữ dội vùng thắt lưng và rễ thần kinh
III. NHỮNG THĂM DÒ PHỤ
Thăm dò cần thiết là chụp cộng hưởng từ (IRM) cột sống, tập trung vào mức thương tổn. Đó là thăm dò duy nhất cho phép chẩn đoán và đặt một chỉ định đình ngoại khoa.
IV. NHỮNG THĂM DÒ KHÁC
– Calcémie : quan trọng để loại bỏ một tăng canxi huyết trong trường hợp di căn cột sống.
– Bilan tiền phẫu : bilan đông máu ; đếm hồng bạch cầu, điện giải đồ máu, nhóm máu. Những chèn ép tủy do di căn có thể gây xuất huyết quan trọng trong lúc mổ cần đặt máu trước khi mổ.
V. ĐIỀU TRỊ
Điều trị trước hết là ngoại khoa. Một corticothérapie có thể là một hỗ trợ điều trị trong những thương tổn ung thư.
Thông tiểu và phòng ngừa escarre phải được thực hiện hệ thống
Reference : Pratique de la réanimation et de la médecine d’urgences
BS NGUYỄN VĂN THỊNH
(15/3/2015)

Cấp cứu lão khoa số 19 – BS Nguyễn Văn Thịnh

SỐT Ở NGƯỜI GIÀ
(FIEVRE RECENTE)

Bertrand François
Professeur de la faculté de médecine de Nice
Pras Pierre
Chef de service de gérontologie clinique
CHU de Nice
Sự xuất hiện một tăng thân nhiệt ở một người già tạo nên một tình huống thường thuộc về cấp cứu vì lẽ khả năng xuất hiện trong một thời hạn rất ngắn những biến chứng tiến triển. Những bệnh nhiễm khuẩn thường gặp ở người già chủ yếu có tình trạng sức khỏe kém. Mức độ nghiêm trọng tiềm tàng là quan trọng có thể ảnh hưởng không những tiên lượng sinh tồn mà còn tiên lượng chức năng.
Những nhiễm trùng vi khuẩn liên quan chủ yếu những đường hô hấp (50%), đường tiểu (25%), hiếm hơn da (17%) và những đường tiêu hóa (4%).
Những nhiễm trùng bệnh viện và cộng đồng đặc biệt đáng quản ngại và nghiêm trọng
Chúng tôi sẽ không đề cập ở đây những sốt kéo dài.

I. CHẨN ĐOÁN
1. NGAY CÚ ĐIỆN THOẠI ĐẦU TIÊN
Đã có thể có một ý tưởng về mức độ cấp cứu và cho vài lời khuyên hữu ích :
– đánh giá về cường độ của sốt, thời gian và tiến triển của nó.
– đánh giá ảnh hưởng của sốt bằng cách tìm kiếm những dấu hiệu nghiêm trọng : khát, trạng thái lả, lú lẫn tâm thần, khó thở, những rối loạn tiêu hóa, ăn mất ngon.
– khuyên tránh mọi điều trị thường nghiệm (traitement empirique), nằm liệt hoàn toàn, khuyên cho uống, dĩ nhiên nếu không có những dấu hiệu tiêu hóa “ngoại khoa”
– khuyên tiếp tục những điều trị hiện tại được đánh giá là cần thiết.
2. Ở NHÀ CỦA BỆNH NHÂN
Phân tích tình cảnh gia đình và môi trường xã hội của bệnh nhân cho phép trả lời một câu hỏi thiết yếu. Có thể điều trị ở nhà hay không ?
Vấn chẩn định rõ :
+ cách xuất hiện : đột ngột hay từ từ
+ ngày tháng xuất hiện và cách tiến triển ;
+ những dấu hiệu kèm theo :
– hoặc định hướng : đường tiểu (thiểu-vô niệu), da, phế quản, tiêu hóa, hay những bệnh khác,
– hoặc mức độ nghiêm trọng : run lạnh dữ dội và lập lại, ra nhiều mồ hôi, mửa, lả người, lú lẫn mới đây, khó thở.
+ Những tiền sử : tares viscérales có trước, mà phải đánh giá nguy cơ trầm trọng thêm.
Những tiền sử này, liên kết với một “fragilité latente” của bệnh nhân tạo điều kiện cho sự bùng nổ hay sự tái phát của một nhiễm trùng : diverticulose colique, adénome prostatique, sỏi đường tiểu hay đường mật, viêm tai giữa mãn tính, soins dentaires hay can thiệp ngoại khoa mới đây, điều trị suy giảm miễn dịch (traitement immunodépresseur).
3. KHÁM LÂM SÀNG
Nhằm tìm kiếm :
a. NHỮNG DẤU HIỆU NGHIÊM TRỌNG
Tình trạng các kết mạc, hạ huyết áp < 9 mmHg, Tim nhịp nhanh > 125 đập/phút, Tần số hô hấp > 30 chu kỳ/phút, những rối loạn vigilance, Nhiệt độ < 35 độ C hay = 40 độ C, những marbrure, hay sự xuất hiện của một ban xuất huyết.
b. NHỮNG DẤU HIỆU ĐỊNH HƯỚNG
Như là dạng vẻ của nước tiểu và đờm
– khám đít và các gót chân để tìm kiếm một vảy mục (escarre).
– ấn chẩn các bắp chân có thể phát hiện một huyết khối tĩnh mạch (thrombose veineuse) ;
– khám trực tràng để tìm kiếm một viêm tiền liệt tuyến ở đàn ông, hay một u phân (fécalome) ;
– khám răng miệng sẽ khám phá những gốc răng còn lại hay một muguet ;
– khám tai để tìm kiếm một viêm tai mãn tính bội nhiễm
Vấn chẩn và khám lâm sàng thường cho phép đánh giá về mức độ cấp cứu của tình huống, và định hướng về nguyên nhân
4. BILAN NGOẠI LÂM SÀNG
Bilan ngoại lâm sàng gồm có :
– numération formule sanguine các tiểu cầu, CRP, VS và một điện di các protéine huyết thanh.
– một bandelette urinaire phát hiện sự hiện diện của những bạch cầu và của nitrite. Sự âm tính làm ít có khả năng một nhiễm trùng đường tiểu (> 98%). Sự dương tính có thể chỉ một nhiễm trùng đường tiểu và với sự hiện diện của sốt phải thực hiện một ECBU :
– một ECBU nước tiểu
– một cặp cấy máu để tìm kiếm một bactériémie ;
– những định lượng créatinine, đường huyết, protidémie, và ionogramme máu để đánh giá những ảnh hưởng chuyển hóa của nhiễm trùng
II. NHỮNG NGUYÊN NHÂN CHÍNH CỦA SỐT
CHẨN ĐOÁN
Trong vài trường hợp chẩn đoán có thể được xác lập chỉ trên những dữ kiện lâm sàng và/hoặc những kết quả của “bilan de débrouillage”.
Tuy nhiên tình huống rất khác nhau tùy theo đó là một bệnh nhân độc lập (autonome) được khám ở nhà hay một cụ già nằm liệt giường trong một cơ sở điều trị.
1. Ở MỘT BỆNH NHÂN ĐỘC LẬP TRÊN BÌNH DIỆN VẬT LÝ
Ta sẽ tìm kiếm trước hết những nhiễm trùng phổi, đường tiểu, tiêu hóa, da, răng.
a. NHIỄM TRÙNG ĐƯỜNG TIỂU
– Symptomatologie d’appel thường nghèo nàn, triệu chứng tiểu rát (brulure urinaire) có thể tối thiểu thậm chí vắng mặt và sốt có thể chỉ là triệu chứng duy nhất.
– Phải tìm kiếm những yếu tố làm dễ, chủ yếu là tuyến tiền liệt ở đàn ông, sinh dục ở phụ nữ, và một tình trạng bón dai dẳng ở cả hai giới.
– Nếu colibacille vẫn là mầm bệnh thường được quy kết, vai trò của Proteus gia tăng ở người cao tuổi nhất và nhất là những người có sức khỏe kém nhất.
– Do đó ECBU với kháng sinh đồ là cần thiết ngay nhiễm trùng đường tiểu lần đầu.
– Những nhiễm trùng đường tiểu kèm theo những triệu chứng đường tiểu phải được điều trị theo cùng những cách thức được áp dụng ở những người trẻ hơn, đồng thời xét đến những quy tắc kê đơn thông thường đối với lứa tuổi này (chức năng thận, các thuốc liên kết…). Những điều trị ngắn ngày không được khuyến nghị ở người già.
– Khi không có những triệu chứng đường tiểu, cái khó là quy trách nhiệm sốt cho vi khuẩn niệu. Sự tìm kiếm một bệnh lý đường tiểu phải được thực hiện một cách hệ thống.
– Nếu vi khuẩn niệu riêng rẻ bởi vì không giải thích được bởi một bệnh lý niệu khoa, quy tắc là không điều trị.
b. NHIỄM TRÙNG PHẾ QUẢN- PHỔI
Chúng tập hợp những viêm phổi, viêm phế quản cấp tính và những bội nhiễm của những viêm phế quản mãn tính. Chúng có thể là do virus hay vi khuẩn. Về những nhiễm trùng do vi khuẩn, phế cầu khuẩn và Haemophilus là những mầm bệnh thường gặp nhất, nhưng tần số của những nhiễm trùng do Klebsiella pneumoniae và do tụ cầu khuẩn vàng gia tăng ở người già.
Một liệu pháp kháng sinh sớm sau khi lấy những mẫu nghiệm vi trùng thường thích hợp hơn so với người trưởng thành
c. NHIỄM TRÙNG TIÊU HÓA
Triệu chứng học có thể gây nhầm lẫn : biến đổi tình trạng tổng quát mà không có dấu hiệu tiêu hóa và phúc mạc. Đó có thể là một viêm túi cùng (diverticulite) (5 đến 10% những người trên 50 tuổi và 80% những người trên 85 tuổi có diverticulose colique) hay một nhiễm trùng đường mật. Khả năng này phải khiến giữ quan sát bệnh nhân trong môi trường bệnh viện. Bệnh nhân phải được cho nhịn ăn với đắp nước đá trên bụng, bù nước bằng đường tĩnh mạch, và điều trị bằng một liệu pháp kháng sinh có kháng khuẩn phổ rộng bằng đường tĩnh mạch.
d. NHIỄM TRÙNG DA
Những nhiễm trùng da khá thường gặp : loét giãn tĩnh mạch (ulcère variqueux) bội nhiễm, viêm tế bào nhiễm trùng (cellulite infectieuse), zona…Chúng có thể do vi khuẩn (chủ yếu liên cầu khuẩn, tụ cầu khuẩn, pyocyanique) hay do nấm. Chúng có thể là điểm xuất phát của những nhiễm trùng huyết.
2. Ở BỆNH NHÂN ĐÃ MẤT TÍNH ĐỘC LẬP
Ta sẽ tìm kiếm những nguyên nhân sau đây :
a. NHIỄM TRÙNG ĐƯỜNG TIỂU
Những yếu tố làm dễ có thể là sự hiện diện hay một tiền sử mới đây đặt ống thông tiểu giữ lưu (sonde à demeure), một sự ứ đọng nước tiểu, một sự ứ đọng phân (stase stercorale) hay một u phân (fécalome). Ở bệnh nhân mang một sonde à demeure, vi khuẩn niệu không triệu chứng (bactériurie asymptomatique) không cần phải điều trị. Kháng sinh liệu pháp chỉ dành cho những đợt nhiễm trùng triệu chứng.
b. TĂNG TIẾT PHẾ QUẢN
Điểm khởi đầu có thể là một fausse route đã gây nên một pneumonie de déglutition. Luôn luôn nghĩ đến phòng ngừa những nhiễm trùng hô hấp do vi khuẩn, ngoài những biện pháp vệ sinh, bằng những tiêm chủng chống cúm và chống phế cầu khuẩn
c. NHỮNG MẢNG MỤC
Những mảng mục (escarre) bị bội nhiễm phải tìm kiếm một cách hệ thống ở những vùng đè ép
d. VIÊM TĨNH MẠCH SÂU VÀ VIÊM TĨNH MẠCH
e. MẤT NƯỚC
Những nguyên nhân chủ yếu do nhiều yếu tố : hạn chế ăn uống, những điều kiện khí hậu, sử dụng lâu dài những thuốc lợi tiểu.
f. NHIỄM KHUẨN HUYẾT
Điểm xuất phát do điều trị (cathéter, ống thông tiểu)
III. KẾT LUẬN
Ở người già sốt phải được xem như là một cấp cứu điều trị. Liệu pháp kháng sinh phải có những chỉ định rộng rãi hơn so với ở người lớn, tuy vậy không được lơ là những thăm dò bổ sung cần thiết. Song song với điều trị kháng sinh, phải đánh giá tình trạng dinh dưỡng và mức cung cấp nước và chú ý duy trì sự thích đáng của những cung cấp với những nhu cầu calorio-protidique của bệnh nhân. Sự theo dõi những tác dụng phụ của kháng sinh liệu pháp và sự tiến triển của một bệnh lý liên kết phải được đảm bảo.
                                      Những cạm bẫy và những đặc điểm
Với bệnh lý y hệt, nhiệt độ ở người già nói chung ít cao hơn so với người trưởng thành trẻ tuổi hơn. Không nên cho rằng một tăng cao nhiệt độ vừa phải a priori hiền tính.
Sự ăn mất ngon và liệt giường làm trầm trọng sự mất calorico-azotée và là nguồn của những mảng mục, càng trầm trọng khi chúng càng kéo dài. Hãy nghĩ chống lại khuynh hướng tự nhiên, bản năng của bệnh nhân già bị sốt : ” Nằm tại giường, nhịn ăn, ủ ấm”
Khi bị sốt, những nhu cầu calorie gia tăng. Sự theo dõi về mặt cung cấp thức ăn là cần thiết.
Tình trạng mất nước và sự hiện diện của những mảng mục là những yêu tố xấu đối với tiên lượng của nhiễm trùng.
Urgences du sujet âgé
BS NGUYỄN VĂN THỊNH
(25/3/2015)

Cấp cứu ngoại thần kinh số 6 – BS Nguyễn Văn Thịnh

CHÈN ÉP TỦY CẤP TÍNH
(COMPRESSION MEDULLAIRE AIGUE)

N.Bruder
Service d’Anesthésie-Réanimation
Hôpital de la Timone, Marseille
Chèn ép tủy bán cấp (compression médullaire subaigue) là một cấp cứu ngoại khoa. Sự nặng thêm về mặt lâm sàng, dẫn đến bại liệt hai chi (paraplégie) hay bại liệt bốn chi (tétraplégie), thường rất nhanh và những khả năng phục hồi phần lớn tùy thuộc vào mức độ nhanh chóng của giải phóng chèn ép tủy (décompression médullaire).

I. LÂM SÀNG
Yếu tố chính là sự xuất hiện dần dần của một thiếu sót cảm giác và vận động mà giới hạn trên chỉ mức thương tổn. Triệu chứng học thường bắt đầu bằng những rối loạn bước, rồi một thiếu sót vận động invalidant xuất hiện và kèm theo những rối loạn cơ vòng (troubles sphinctériens). Trong những thương tổn vùng thắt lưng thấp (lésions lombaires basses), triệu chứng học có thể chỉ là một hội chứng đuôi ngựa (syndrome de la queue de cheval). Những dấu hiệu này có thể được đi trước bởi những triệu chứng đau ở vùng lưng hay cổ, đau rễ thần kinh (douleur radiculaire). Đau và yếu cơ có thể kéo dài nhiều tuần nhưng sự trở nặng thành liệt hai chi hay liệt 4 chi có thể xuất hiện trong vài giờ.
II. CĂN NGUYÊN
1. CHẤN THƯƠNG
Khi xảy ra một chấn thương, chẩn đoán hiển nhiên khi hiện hữu tức thời một thiếu sót thần kinh. Thiếu sót này có thể xuất hiện thứ phát trong những thương tổn không vững (lésions instables) không được nhận thấy, nhất là ở cổ, hay khi xuất hiện một thoát vị đĩa đè ép (hernie discale compressive).
2. ĐÈ ÉP DO UNG THƯ (COMPRESSION NEOPLASIQUE)
Đó là nguyên nhân thường gặp nhất. Đè ép tủy xảy ra trong những di căn cột sống của các ung thư (nhất là ung thư phổi, vú, tiền liệt tuyến, thận và tuyến giáp), trong những syndrome myélo-prolifératifs (myélome, lymphome, ung thư bạch cầu).
3. MÁU TỤ NGOÀI MÀNG CỨNG (HEMATOME EPIDURAL)
Phải được gợi lên ở một bệnh nhân đang điều trị kháng đông hay bị những rối loạn đông máu, sau khi chọc dò tủy sống, hay sau khi mổ cột sống (chirurgie rachidienne). Trong nguyên nhân này thường có đau dữ dội.
4. ÁP XE NGOÀI MÀNG CỨNG (ABCES EPIDURAL)
Có thể do là biến chứng của viêm xương tủy (ostéomyélite). Phải gợi lên khi có sốt, nhất là một động tác ngoại khoa hay một chọc dò đã được thực hiện nhiều tháng trước. Triệu chứng học thường được đi trước bởi đau dữ dội vùng thắt lưng và rễ thần kinh
III. NHỮNG THĂM DÒ PHỤ
Thăm dò cần thiết là chụp cộng hưởng từ (IRM) cột sống, tập trung vào mức thương tổn. Đó là thăm dò duy nhất cho phép chẩn đoán và đặt một chỉ định đình ngoại khoa.
IV. NHỮNG THĂM DÒ KHÁC
– Calcémie : quan trọng để loại bỏ một tăng canxi huyết trong trường hợp di căn cột sống.
– Bilan tiền phẫu : bilan đông máu ; đếm hồng bạch cầu, điện giải đồ máu, nhóm máu. Những chèn ép tủy do di căn có thể gây xuất huyết quan trọng trong lúc mổ cần đặt máu trước khi mổ.
V. ĐIỀU TRỊ
Điều trị trước hết là ngoại khoa. Một corticothérapie có thể là một hỗ trợ điều trị trong những thương tổn ung thư.
Thông tiểu và phòng ngừa escarre phải được thực hiện hệ thống
Reference : Pratique de la réanimation et de la médecine d’urgences
BS NGUYỄN VĂN THỊNH
(15/3/2015)

Cấp cứu bệnh nhiễm khuẩn số 12 – BS Nguyễn Văn Thịnh

NGỘ ĐỘC THỨC ĂN
(FOOD POISONING)

Jeff S. Beckman, M.D.
Department of Emergency Medicine
Denver Health Medical Center
Denver, Colorado
1/ KỂ NHỮNG NGUYÊN NHÂN CỦA NGỘ ĐỘC THỨC ĂN
– Ngoại độc tố được sản xuất bởi vi sinh vật
– Các vi sinh vật (vi khuẩn, nấm, virus, ký sinh trùng) (được xếp loại thành xâm nhập (invasive), không xâm nhập (noninvasive) hoặc sinh độc tố (toxin-producing)
– Các chất hiện diện trong thức ăn một cách tự nhiên (amatoxin (nấm), dinoflagellates hay thallophytes)

2/ LIỆT KÊ NHỮNG TRIỆU CHỨNG THÔNG THƯỜNG CỦA NGỘ ĐỘC THỨC ĂN
– Nôn mửa
– Tiêu chảy
– Sốt nhẹ
– Đau bụng quặn
3/ BỆNH SỬ NÀO LÀM NGHĨ ĐẾN NGỘ ĐỘC THỨC ĂN ?
Sự hiện diện của những triệu chứng tương tự ở những thành viên khác trong gia đình hoặc những người khác đã ăn cùng thức ăn hay uống cùng thứ nước, hiện diện máu trong phân, và mới đây đi du lịch ở các nước kém phát triển hoặc ở vùng núi. Những điểm khác trong bệnh sử bao gồm tiến triển của bệnh (cấp tính, bán cấp tính, mãn tính), thức ăn vừa mới được ăn vào, những bệnh lý nội khoa khác (tình trạng HIV, cao huyết áp), các phẫu thuật bụng trước đây, bệnh sử về hoạt động tình dục và các động vật nuôi trong nhà.
4/ CÁC DẤU HIỆU VẬT LÝ THÔNG THƯỜNG ?
Nói chung bụng mềm và rất ít nhạy cảm đau (tender) khi ấn chẩn, không có dấu hiệu phúc mạc (peritoneal signs). Nhu động ruột nghe được (bowel sounds) có thể tăng hoạt động hoặc bình thường, và máu có thể hiện diện trong phân.
5/ CÁC DẤU HIỆU VẬT LÝ NÀO LÀ KHÔNG ĐIỂN HÌNH TRONG NGỘ ĐỘC THỨC ĂN ?
Các vùng bụng nhạy cảm đau (abdominal tenderness) nằm riêng rẻ, các dấu hiệu phúc mạc, phân có máu rõ rệt hoặc đại tiện máu đen (melena) là những dấu hiệu cảnh cáo cần phải thăm dò hơn nữa, bao gồm hội chẩn ngoại khoa hoặc chuyên khoa tiêu hóa. Sốt cao không phải là một đặc tính thông thường của tiêu chảy do vi khuẩn (bacterial diarrhea) ngoại trừ bệnh Shigella (shigellosis).
6/ CHẨN ĐOÁN PHÂN BIỆT TIÊU CHẢY GÂY NÊN BỞI VI TRÙNG XÂM NHẬP VÀ KHÔNG XÂM NHẬP
Các vi khuẩn xâm nhập niêm mạc (Escherichia coli, Salmonella, Shigella, Campylobacter và Yersinia) gây nên tiêu chảy với phân chứa niêm dịch (mucoid) và có máu. Các nội độc tố (enterotoxin) được sản xuất bởi các vi sinh vật khác (siêu vi, Vibrio cholerae, E.Coli, Staphylococcus aureus, Clostridium perfringens, Clostridium difficile, và Bacillus cereus) ảnh hưởng lên bơm cyclic adenosine monophosphate nằm trong niêm mạc ruột và gây nên tiêu chảy toàn nước (watery diarrhea).
7/ MÔ TẢ ĐIỀU TRỊ KHỞI ĐẦU TRÚNG ĐỘC THỨC ĂN Ở PHÒNG CẤP CỨU.
Khi bệnh sử và khám vật lý đã được thực hiện và chẩn đoán là viêm dạ dày-ruột (gastroenteritis) bất kể là nguyên nhân gì, thì mức độ mất nước (dehydration) phải được đánh giá và điều trị. Cần đặc biệt thận trọng ở nhũ nhi và trẻ em ; bệnh tiêu chảy vẫn tiếp tục là một nguyên nhân dẫn đầu về tử vong của trẻ nhỏ trên toàn thế giới, kể cả Hoa Kỳ. Có thể cần phải bồi hoàn nước bằng đường tĩnh mạch với dung dịch crystalloid. Bồi hoàn nước bằng đường miệng (oral rehydration) được thực hiện bằng những dung dịch chứa glucose đẳng trương (giúp làm dễ hấp thụ nước và sodium). Có thể cho các thuốc chống nôn (emetics).
8/ XÉT NGHIỆM TỐT NHẤT ĐỂ CHẨN ĐOÁN BỆNH TIÊU CHẢY XÂM NHẬP (INVASIVE DIARRHEA) ?
Sự hiện diện bạch cầu và máu ẩn (occult blood) trong phân là dấu hiệu tiên đoán nhạy cảm và đặc hiệu về nguyên nhân vi khuẩn xâm nhập niêm mạc (invasive bacteria) của chứng tiêu chảy và khả năng cấy phân dương tính với vi khuẩn.
9/ MONTEZUMA’s REVENGE LÀ GÌ ?
Hầu hết các trường hợp tiêu chảy của khách du lịch (traveler’s diarrhea, turista hay Montezuma ‘s revenge) xuất phát từ Mexico, Nam Mỹ hoặc châu Phi. Trong trường hợp điển hình, tiêu chảy toàn nước bắt đầu vài ngày sau du lịch và thông thường nhất gây nên bởi enterotoxigenic E.coli.
E.coli có thể gây ỉa chảy bằng ba cơ chế :
(1) Giống sinh nội độc tố (enterotoxigenic strains) sản xuất một độc tố gây nên tiêu chảy toàn nước, tương tự với tiêu chảy trong bệnh dịch tả.
(2) Giống sinh bệnh ruột (enteropathogenic strains) sinh sống ở ruột, gây nên những đợt tiêu chảy trong các nhà trẻ bệnh viện.
(3) Giống xâm nhập niêm mạc ruột (enteroinvasive strains) (E.coli 0157 : H7) có thể gây nên bệnh giống shigella với phân có máu và niêm dịch.
10/ MÔ TẢ ĐIỀU TRỊ BỆNH TIÊU CHẢY CỦA KHÁCH DU LỊCH.
Dịch và các chất điện giải, cùng với các tác nhân làm giảm thể tích phân là trụ cột của điều trị chứng tiêu chảy của khách du lịch. Bismuth subsalicylate (Pepto-Bismol) làm giảm 50% số lần phân chưa thành hình ; loperamide (Imodium) làm giảm tiêu chảy 80%. Đối với các bệnh nhân không đáp ứng đối với các biện pháp này, hãy xét đến việc điều trị bằng kháng sinh.Trimethoprim-sulfamethoxazole (Bactrim) với liều lượng mạnh gấp đôi (160mg trimethoprim và 800mg sulfamethoxazole), một viên hai lần mỗi ngày, được khuyên sử dụng đối với các khách du lịch đến các vùng không phải bờ biển của Mexico trong mùa hè. Đối với các vùng khác và vào các mùa khác, loại kháng sinh fluoroquinolone là thuốc được lựa chọn. Ciprofloxacin (Ciproxine) (750mg), levofloxacin (Tavanic) (500mg), norfloxacin (Zoroxin) (800mg), azithromycin (Zitromax) (1000mg) có thể cho với liều lượng duy nhất ở phòng cấp cứu hoặc với thời gian điều trị 3 ngày cho những bệnh nhân bị bệnh nặng hơn.
11/ VAI TRÒ ĐIỀU TRỊ PHÒNG NGỪA CHỐNG LẠI TIÊU CHẢY CỦA KHÁCH DU LỊCH ?
– Những kháng sinh phòng ngừa có thể thích hợp đối với những bệnh nhân có tình trạng sức khỏe kém đi du lịch ở những vùng có nguy cơ cao (high-risk areas). Các bệnh nhân với bệnh viêm ruột (inflammatory bowel disease), bệnh đái đường phụ thuộc insulin, bệnh tim ở các người già hoặc AIDS, có thể xét đến việc điều trị bằng kháng sinh phòng ngừa, sử dụng trimethoprim-sulfamethoxazole hay một fluoroquinolone, tùy thuộc vùng mà họ đến để du lịch. Các khách du lịch không có bệnh tật và muốn được phòng ngừa, có thể khuyên sử dụng bismuth subsalicylate (Pepto-Bismol) với liều lượng 524 mg, 4 lần mỗi ngày. Không nên cho dược phẩm này ở các bệnh nhân đang được điều trị bằng thuốc chống đông máu (anticoagulants) hoặc salicylates hoặc ở những người dị ứng với salicylates.
12/ VI KHUẨN GÂY TIÊU CHẢY NÀO LIÊN KẾT VỚI CO GIẬT DO SỐT Ở TRẺ EM ?
Nhiễm trùng Shigella ở các trẻ nhỏ thường gây nên sốt cao với các cơn co giật do sốt (febrile seizures) trước khi khởi đầu bệnh tiêu chảy. Bệnh có thể rất nặng và gây tử vong do mất nước (dehydration), có thể xảy ra trong vòng 8 giờ kể từ khi bắt đầu tiêu chảy. Đau bụng và sốt thường đi trước tiêu chảy.Trong trường hợp điển hình, phân có máu và có dạng nhầy (mucoid). Xét nghiệm máu cho thấy bạch cầu tăng cao và chuyển sang trái rõ rệt.
13/ MÔ TẢ MỘT BỆNH NHÂN TRÚNG ĐỘC THỨC ĂN DO SALMONELLA.
Trong trường hợp điển hình, bệnh nhân đi tiêu chảy toàn nước và đau bụng quặn, thường không có nôn mửa, xảy ra khoảng 12 đến 36 giờ sau khi ăn đồ ăn bị ô nhiễm. Các nguồn thức ăn gồm có trứng, gà vịt, thịt, gà tây và sửa hoặc nước trái cây không được tiệt trùng. Nếu như có nhiễm khuẩn huyết (bacteremia), các triệu chứng toàn thân như sốt, ho, đau đầu và viêm màng não giả (meningism) có thể xảy ra. Nên nhập viện và điều trị trường hợp sốt ruột (enteric fever).
14/ VI KHUẨN NÀO TĂNG SINH TRONG THỊT ĐƯỢC NẤU TRƯỚC HOẶC ĐƯỢC HÂM LẠI KHÔNG KỸ VÀ GÂY NÊN ĐAU BỤNG QUẶN VÀ TIÊU CHẢY 12 GIỜ SAU KHI ĂN THỨC ĂN BỊ Ô NHIỄM ?
– Clostridium perfringens thường liên kết với các cơ sở phục vụ ăn uống, nơi thức ăn được nấu trước. Những triệu chứng thông thường gồm có tiêu chảy, đau bụng và đôi khi nôn. Không có sốt và mửa.
15/ NGỘ ĐỘC THỨC ĂN NÀO GÂY TRIỆU CHỨNG TRONG VÒNG 1 ĐẾN 6 GIỜ ?
– Nội độc tố staphylococcus aureus bền với nhiệt ; gây nôn mửa, đau bụng quặn và tiêu chảy nhẹ. Nội độc tố này được sản xuất trong thịt làm nguội, thực phẩm, thức ăn bơ sữa, hàng bánh mì được để trong thời gian lâu ở nhiệt độ của phòng
– Nội độc tố bacillus cereus được tìm thấy trong cơm và gây nên nôn mửa nhiều.
16/ NHIỄM ĐỘC THỨC ĂN DO VI KHUẨN THÔNG THƯỜNG NHẤT Ở HOA KỲ ?
– Nhiễm trùng bởi Campylobacter jejuni gây đau bụng với tiêu chảy và trong 1/3 trường hợp có tiền triệu giống bệnh cúm trước khi bắt đầu các triệu chứng tiêu hóa. Phần lớn được tìm thấy trong thịt gia cầm sống hoặc không được nấu kỹ, C.jejuni có thể gây nên một hội chứng Guillain Barré hoặc viêm khớp phản ứng (reactive arthritis) trong vòng 12 giờ sau khi bị nhiễm trùng.
17/ BỆNH NHIỄM TRÙNG THỨC ĂN NÀO CÓ THỂ CÓ TRIỆU CHỨNG GIỐNG VỚI VIÊM RUỘT THỪA CẤP TÍNH Ở TRẺ LỚN VÀ NGƯỜI TRƯỞNG THÀNH ?
– Nhiễm trùng bởi Yersinia enterocolitica có thể dẫn đến những triệu chứng cấp tính như đau vùng hố chậu phải, sốt cao, nôn mửa và tăng bạch cầu với tiêu chảy nhẹ. Hội chứng này, được gọi là viêm ruột thừa giả (pseudoappendicitis) hay viêm hạch mạc treo (mesenteric adenitis) đã dẫn đến nhiều trường hợp cắt bỏ ruột thừa không cần thiết.
18/ SCOMBROID LÀ GÌ ?
– Ngộ độc bởi histamine của cá (histamine fish poisoning) (hay scombroid ) được gây nên do sự tạo thành histamine hay amine trong cá ngừ California (tuna), cá xanh (bluefish), cá thu (mackerel), cá marlin (poisson épieu). Các triệu chứng đỏ bừng mặt (flushing), mày đay (urticaria), chóng mặt và dị cảm (paresthesia) xảy ra từ nhiều phút đến nhiều giờ sau khi ăn thức ăn bị ô nhiễm. Hầu hết các trường hợp tự hạn chế và diphenhydramine và cimetidine có thể được sử dụng để làm giảm triệu chứng. Một số bệnh nhân có thể bị độc tính tương đương với quá mẫn cấp tính (acute anaphylaxis) và cần được xử lý tích cực hơn.
19/ KÝ SINH TRÙNG NÀO NÊN ĐƯỢC NGHI NGỜ Ở MỘT NGƯỜI ĐI CẮM TRẠI BỊ TRƯỚNG BỤNG KINH NIÊN VÀ TIÊU CHẢY LUÂN PHIÊN VỚI TÁO BÓN.
Giardia lamblia gây dịch địa phương (endemic) ở nhiều vùng ở Hoa Kỳ và là ký sinh trùng quan trọng nhất của nước Mỹ. Nhiễm trùng mắc phải do uống nước bị ô nhiễm hoặc tiếp xúc từ người này qua người khác (person-to-person contact). Trên toàn bộ dân số, có 7% người mang ký sinh trùng ở giai đoạn nang (cystic stage). Thường được điều trị hiệu quả với metronidazole, mặc dầu việc sử dụng thuốc trong điều trị bệnh nhiễm trùng này không được ưng thuận.
20/ CÓ NÊN SỬ DỤNG KHÁNG SINH ĐỂ ĐIỀU TRỊ BỆNH TIÊU CHẢY DO VI KHUẨN HAY KHÔNG ?
Việc sử dụng kháng sinh trong điều trị bệnh tiêu chảy cấp tính gây tranh cải và kháng sinh không được chỉ định đối với những trường hợp tiêu chảy không xâm nhập niêm mạc (noninvasive diarrhea). Bất cứ kháng sinh nào cũng có khả năng đưa lại sự tăng sinh vượt mức của Clostridium difficile, dẫn dến bệnh viêm đại tràng giả mạc (pseudomembranous colitis). Điều trị kháng sinh bệnh tiêu chảy ra máu (bloody diarrhea) nói chung không được khuyến nghị do sự liên kết với nhiễm trùng bởi E.Coli 0157 : H7.Trong những nhiễm trùng này cac kháng sinh không cải thiện đáng kể tiên lượng, và có thể đẩy mạnh sự phát triển hội chứng tan máu tăng urée (hemolytic-uremic syndrome), do làm gia tăng sự phân giải của các vi khuẩn với sự phóng thích của nội độc tố và độc tố giống Shiga (Shiga-like toxin). Những người trưởng thành có bệnh tiêu chảy với thời gian dưới hai tuần, sốt và có bạch cầu trong phân (fecal keucocyte ) hay phân dương tính với Hemoccult, có thể được điều trị trong 3 đến 5 ngày với trimethoprim-sulfamethoxazole (1viên, 2 lần mỗi ngày), ciprofloxacin (500mg, hai lần mỗi ngày), norfloxacin (400mg, hai lần mỗi ngày), levofloxacin (500mg, mỗi lần mỗi ngày) hoặc azithromycin (500mg mỗi lần mỗi ngày).
21/ CÓ NÊN ĐIỀU TRỊ GIẢM TRIỆU CHỨNG BẰNG CÁC THUỐC CẦM TIÊU CHẢY KHÔNG ?
Điều trị giảm triệu chứng bằng các thuốc cầm tiêu chảy chỉ được đề nghị cho những bệnh nhân với tiêu chảy không xâm nhập niêm mạc (noninvasive diarrhea) (không có sự hiện diện của bạch cầu trong các kính phết phân). Các bệnh nhân với tiêu chảy xâm nhập niêm mạc (invasive diarrhea) có thể phát triển chứng to đại tràng do độc tố (toxic megacolon) và tiếp tục thải các vi khuẩn nếu được điều trị bằng các thuốc chống lại nhu động (antimotility drugs). Tuy nhiên, mới đây, loperamide (Imodium) đã được sử dụng một cách an toàn cho tất cả các dạng viêm dạ dày-ruột (gastroenteritis), nhưng với diphenoxylate (Lomotil) thì nên tránh trong trường hợp tiêu chảy xâm nhập niêm mạc.
22/ XỬ TRÍ NÀO LÀ THÍCH HỢP ĐỐI VỚI MỘT BỆNH NHÂN BỊ NGỘ ĐỘC THỨC ĂN ?
Hầu hết các bệnh nhân có thể được cho về nhà với chỉ thị tránh những thức ăn chứa lactose cho đến khi tiêu chảy ngừng lại. Phải cho những chỉ thị về sự sử dụng thích hợp những dịch trong sạch và tăng dần trong chế độ ăn uống như thế nào. Sự theo dõi phải được sắp xếp nếu những triệu chứng kéo dài trong 1 tuần. Hãy đảm bảo loại bỏ những nguyên nhân khác gây mửa và ỉa chảy ở người già. Theo dõi sát để đánh giá tình trạng mất nước là chủ yếu ở các nhũ nhi nhỏ.
Emergency Medicine Secrets. Third Edition. 2003
BS NGUYỄN VĂN THỊNH
(28/11/2007)
(Update 3/3/2012)
Review 25/3/2015